SlideShare ist ein Scribd-Unternehmen logo
1 von 53
Unit-3 FREQUAENCY DISTRIBUTION
RAI UNIVERSITY, AHMEDABAD 1
Course: MCA
Subject: Computer Oriented Numerical
Statistical Methods
Unit-3
RAI UNIVERSITY, AHMEDABAD
Unit-3 FREQUAENCY DISTRIBUTION
RAI UNIVERSITY, AHMEDABAD 2
Unit-III- Frequaency Distribution
Sr.
No.
Name of the Topic Page
No.
1 Introduction, Collection of data, Classification of data 2
2 Introduction to frequency distribution , Class Limit, , Class
Interval,Class frequency, Class mark, Class Boundaries, Width of a
class
5
3 Frequency density, Relative frequency, Percentage frequency,
Cumulative frequency
9
4 Introduction, Arithmetic Mean, Mean forDiscrete frequency
distribution, Mean for Continuous frequency distribution,Weighted
Arithmetic Mean
11
5 Properties of A.M., Merits & De merits of A.M., 22
6 Median for raw data, Discrete frequency distribution, Continuous
frequency distribution,
23
7 Merits and demerits of Median 26
8 Mode for raw data, D.f.s., c.f.s., 27
9 Merits & demerits of mode 30
10 Introduction, Range, coefficient of range,Merit & Demerit of
Range
30
11 Quartiles, Quartiles deviations, coefficient of quartile deviations 31
12 Mean deviation and coefficient of mean deviation,Merit and
Demerit of Mean Deviation
33
13 Standard Deviation and variance for all types of frequency
distribution
38
14 Coefficient of variation 48
Unit-3 FREQUAENCY DISTRIBUTION
RAI UNIVERSITY, AHMEDABAD 3
1.1 Introduction:
A sequence of observation, made on a set of objects included in the sample drawn
from population is known as statistical data.
(1). Ungrouped data
Data which have been arranged in a systematic order are called raw data or
ungrouped data.
(2) .Grouped data
Data presented in the form of frequency distribution is called grouped data.
1.2Collectionof Data:
The first step in any enquiry (investigation) is collection of data. The data may be
collected for the whole population or for a sample only. It is mostly collected on
sample basis. Collection of data is very difficult job. The enumerator or
investigator is the well trained personwho collects the statistical data. The
respondents (information) are the persons whom the information is collected.
1.3 ClassificationofData:
Data classification is the process oforganizing data into categories for its most
effective and efficient use.
A well-planned data classification system makes essential data easy to find and
retrieve.
Written procedures and guidelines for data classification should define what
categories and criteria the organization will use to classify data and specify the
roles and responsibilities of employees within the organization regarding data
stewardship. Once a data-classification scheme has been created, security standards
that specify appropriate handling practices for each category and storage standards
that define the data's lifecycle requirements should be addressed.
Here is an example of what a data classification scheme might look like:
Unit-3 FREQUAENCY DISTRIBUTION
RAI UNIVERSITY, AHMEDABAD 4
Category4: Highly sensitive corporateand customer data that if disclosed could
put the organization at financial or legal risk.
Example: Employee social security numbers, customer credit card numbers
Category3: Sensitive internal data that if disclosed could negatively affect
operations.
Example: Contracts with third-party suppliers, employee reviews
Category2: Internal data that is not meant for public disclosure.
Example: Sales contest rules, organizational charts
Category1: Data that may be freely disclosed with the public.
Example: Contact information, price lists
1.3.1 Types of Data:
There are two types (sources) for the collection of data.
(1) Primary Data (2) SecondaryData
(1) Primary Data:
The primary data are the first hand information collected, compiled and published
by organization for some purpose. They are most original data in character and
have not undergone any sortof statistical treatment.
Example: Population census reports are primary data because these are collected,
complied and published by the population census organization.
(2) SecondaryData:
The secondarydata are the second hand information which are already collected by
some one (organization) for some purposeand are available for the present study.
The secondarydata are not pure in character and have undergone some treatment at
least once.
Example: Economics survey of England is secondarydata because these are
collected by more than one organization like Bureau of statistics, Board of
Revenue, the Banks etc…
Unit-3 FREQUAENCY DISTRIBUTION
RAI UNIVERSITY, AHMEDABAD 5
1.3.2DifferencebetweenPrimary and SecondaryData:
The difference between primary and secondary data is only a change of hand. The
primary data are the first hand data information which is directly collected form
one source. They are most original data in character and have not undergone any
sort of statistical treatment while the secondarydata are obtained from some other
sources or agencies. They are not pure in character and have undergone some
treatment at least once.
For Example: Supposewe interested to find the average age of MS students. We
collect the age’s data by two methods; either by directly collecting from each
student himself personally or getting their ages from the university record. The
data collected by the direct personal investigation is called primary data and the
data obtained from the university record is called secondary data.
1.3.3 Types of Classification:
1. Geographical Classification:
i.e. Area wise, e.g. cities, districts, etc.
2. Chronological Classification:
i.e. on the basis of time
3. Qualitative Classification:
i.e. according to some attributes
4. Quantitative Classification:
i.e. in terms of magnitudes
Quantitative classification refers to the classification of data according to some
characteristics that can be measured, such as height, weight, income, sales, profits
etc.
In this type of classification , there are two elements namely
(i). the variable
(ii). The frequency: Frequency is how often something occurs.
Unit-3 FREQUAENCY DISTRIBUTION
RAI UNIVERSITY, AHMEDABAD 6
2.1Definition:frequency Distribution
By counting frequencies we can make a Frequency Distribution table.
There are two types of frequency distribution:
(a). Discrete frequency distribution.
(b). Continuous frequency Distribution.
Example: Newspapers
These are the numbers of newspapers sold at a local shop over the last 10 days:
22, 20, 18, 23, 20, 25, 22, 20, 18, 20
Let us count how many of each number there is:
Here Table-A is a example of discrete frequency distribution
Table-B is a example of continuous frequency distribution.
TABLE-A
Papers Sold Frequency
18 2
19 0
20 4
21 0
22 2
23 1
24 0
25 1
It is also possible to group the values. Here they are in grouped :
Table-B
Unit-3 FREQUAENCY DISTRIBUTION
RAI UNIVERSITY, AHMEDABAD 7
Papers Sold Frequency
15-19 2
20-24 7
25-29 1
2.2ClassLimit:
Class limits are the smallest and largest observations (data, events etc) in each
class. Therefore, each class has two limits: a lower and upper.
Example for, In the above Table-B
For the first class 15-19
The Lower class limit is =15
The Upper class limit is = 19
2.3Class Interval:
The difference between the upper and lower limit of a class is known as class
interval of that class.
Table-B
Papers Sold Frequency
15-19 2
20-24 7
25-29 1
For example: ForTable-B in the class 15-19 the class interval = 4
(i.e. 19-15)
A simple formula to obtain the estimate of appropriate class interval,
i.e. 𝑖 =
𝐿−𝑆
𝑘
where 𝐿 = largest item, S= Smallest item, K = no. of classes
Unit-3 FREQUAENCY DISTRIBUTION
RAI UNIVERSITY, AHMEDABAD 8
2.3.1 Example:If the salary of 100 employees in a commercialundertaking
varied between Rs.10,000and Rs. 30,000and we want to form 10 classes,then
Find the class interval.
Solution: 𝑖 =
𝐿−𝑆
𝑘
 Here 𝐿 = 30,000, 𝑆 = 10,000, 𝑘 = 10
 𝑖 =
30,000−10,000
10
=
20,000
10
= 2000
2.4 Class Frequency:
The number of observations correspondingto a particular class is known as the
frequency of that class or the class frequency.
Table-B
Papers Sold Frequency
15-19 2
20-24 7
25-29 1
For example,
In the Table-B the class 20-24 has frequency 7.
i.e there are 7 days in which no. of papers sold is between 20-24.
If we add together frequencies of all individual classes, we obtain the total
frequency.
The total frequency of Table-B = 2+7+1=10
2.5 Class mark or class mid point:
It is the value lying half-way between the lower and upper class limits of a class
interval.
Unit-3 FREQUAENCY DISTRIBUTION
RAI UNIVERSITY, AHMEDABAD 9
∴ 𝑐𝑙𝑎𝑠𝑠 𝑚𝑎𝑟𝑘 =
𝑈𝑝𝑝𝑒𝑟 𝑙𝑖𝑚𝑖𝑡 𝑜𝑓 𝑡ℎ𝑒 𝑐𝑙𝑎𝑠𝑠+𝑙𝑜𝑤𝑒𝑟 𝑙𝑖𝑚𝑖𝑡 𝑜𝑓 𝑡ℎ𝑒 𝑐𝑙𝑎𝑠𝑠
2
2.6 Class Boundaries:
Class Boundaries are the midpoints between the upper class limit of a class and the
lower class limit of the next class in the sequence. Therefore, each class has an
upper and lower class boundary.
For Example: Table-B
Papers Sold Frequency
15-19 2
20-24 7
25-29 1
For the first class in table-B , 15 – 19
The lower class boundary is the midpoint between 14 and 15, that is 14.5
The upper class boundary is the midpoint between 19and 20, that is 19.5
2.7 Width of a Class:
Difference between two consecutive lower class limits
Difference between two consecutive upper class limits
Example for, Table-B
Papers Sold Frequency
15-19 2
20-24 7
25-29 1
Difference between two consecutive lower class limits
20-15 = 5
Unit-3 FREQUAENCY DISTRIBUTION
RAI UNIVERSITY, AHMEDABAD 10
Difference between two consecutive upper class limits
24-19 = 5
∴Class width=5
3.1Frequency Density:
𝐹𝑟𝑒𝑞𝑢𝑒𝑛𝑐𝑦 𝑑𝑒𝑛𝑠𝑖𝑡𝑦 =
𝑓𝑟𝑒𝑞𝑢𝑒𝑛𝑐𝑦
𝑐𝑙𝑎𝑠𝑠 𝑤𝑖𝑑𝑡ℎ
Frequency density is use to draw histogram .
The following table shows the ages of 25 children on a school bus:
Age Frequency
5-10 6
11-15 15
16-17 4
> 17 0
To draw the histogram we need frequency density:
For class 5-10
𝐹𝑟𝑒𝑞𝑢𝑒𝑛𝑐𝑦 𝑑𝑒𝑛𝑠𝑖𝑡𝑦 =
𝑓𝑟𝑒𝑞𝑢𝑒𝑛𝑐𝑦
𝑐𝑙𝑎𝑠𝑠 𝑤𝑖𝑑𝑡ℎ
=
6
6
= 1
For class 11-15
𝐹𝑟𝑒𝑞𝑢𝑒𝑛𝑐𝑦 𝑑𝑒𝑛𝑠𝑖𝑡𝑦 =
𝑓𝑟𝑒𝑞𝑢𝑒𝑛𝑐𝑦
𝑐𝑙𝑎𝑠𝑠 𝑤𝑖𝑑𝑡ℎ
=
15
5
= 3
Unit-3 FREQUAENCY DISTRIBUTION
RAI UNIVERSITY, AHMEDABAD 11
3.2 Relative frequency:
Relative frequency is the ratio of the number of times an event occurs to the
number of occasions on which it might occurin the same period.
In other words, how often something happens divided by all outcomes.
Example: if your team has won 9 games from a total of 12 games played:
* the Frequency of winning is 9
* the Relative Frequency of winning is
9
12
= 75%
3.3 Percentagefrequency:
Percentage frequency that means calculate percentage of given frequency.
Percentage frequency=
100×𝑓𝑟𝑒𝑞𝑢𝑒𝑛𝑐𝑦 𝑜𝑓 𝑔𝑖𝑣𝑒𝑛 𝑐𝑙𝑎𝑠𝑠
𝑡𝑜𝑡𝑎𝑙 𝑓𝑟𝑒𝑞𝑢𝑒𝑛𝑐𝑦
Papers Sold Frequency
15-19 2
20-24 7
25-29 1
Forclass 15-19 frequency is = 2
∴ Percentage frequency for class 15-19 =
100×2
10
= 20%
Unit-3 FREQUAENCY DISTRIBUTION
RAI UNIVERSITY, AHMEDABAD 12
3.4Cumulative frequency:
The total of a frequency and all frequencies so far in a frequency distribution.
It is the 'running total' of frequencies.
4.1Introduction:
A measure of central tendency is a single value that attempts to describe a set of
data by identifying the central position within that set of data. Measures of central
tendency are sometimes called measures of central location. They are also classed
as summary statistics. The mean (often called the average) is most likely the
measure of central tendency that you are most familiar with, but there are others,
such as the median and the mode.
The mean, median and mode are all valid measures of central tendency.
4.2Arithmetic Mean:
The most popular and widely used measure of representing the entire data by one
value is what most laymen call an average and what the statisticians call the
Unit-3 FREQUAENCY DISTRIBUTION
RAI UNIVERSITY, AHMEDABAD 13
arithmetic mean. Its value is obtained by adding together all the items and by
dividing this total by the number of items.
Arithmetic mean may either be
i. Simple arithmetic mean, or
ii. Weighted arithmetic mean
First of all we have to discuss about Simple Arithmetic Mean.
There are two method for finding simple arithmetic Mean :
1. Direct Method
2. Short-cut Method
4.2.1 DirectMethod for finding Arithmetic Mean :
𝑿̅ =
𝑿 𝟏+𝑿 𝟐+𝑿 𝟑+⋯……+𝑿 𝒏
𝑵
=
∑ 𝑿
𝑵
Here 𝑋̅ = Arithmetic Means
∑ 𝑋 = Sum of all the values of the variable 𝑋
𝑁 = Number of the Observations
Steps for finding Arithmetic mean :
1. Add together all the values of the variable 𝑋 and obtain the total
i.e., ∑ 𝑋
2. Divide this total by the number of observations, i.e., 𝑁
Example-The following table data is the monthly income (in Rs.)of 10
employees in an office:
14780,15760,26690,27750,24840,24920,16100,17810,27050,26950
Calculate the arithmetic mean of incomes.
Solution: Here 𝑁 = 10
 ∴ 𝑋̅ =
𝑋1+𝑋2+𝑋3+⋯……+𝑋 𝑛
𝑁
=
∑ 𝑋
𝑁
Unit-3 FREQUAENCY DISTRIBUTION
RAI UNIVERSITY, AHMEDABAD 14
 ∴ 𝑋̅ =
14780+15760+26690+27750+24840+24920+16100+17810+27050+26950
10
 ∴ 𝑋̅ =
2,22,650
10
 ∴ 𝑋̅ = 22,265
 Hence the Average Income is Rs.22,265
4.2.2 Short-cut Method for finding Arithmetic Mean:
The arithmetic mean can be calculated by using what is known as an arbitrary
Origin. When deviations are taking from an arbitrary origin, the formula for
calculating arithmetic mean is
𝑋̅ = 𝐴 +
∑ 𝑑
𝑁
Where A is the assumed mean and d is the deviation of items from assumed mean,
i.e.,𝑑 = (𝑋 − 𝐴)
Steps for finding Arithmetic mean by shortcut Method
1. Take an assumed mean.
2. Take the deviations of items from the assumed mean and denote these
deviations by 𝑑.
3. Obtain the sum of these deviations, i.e., ∑ 𝑑
4. Apply the formula 𝑋̅ = 𝐴 +
∑ 𝑑
𝑁
Example-- The following table data is the monthly income (in Rs.)of 10
employees in an office:
14780,15760,26690,27750,24840,24920,16100,17810,27050,26950
Calculate the arithmetic mean of incomes by using short cut method.
Solution:
Supposeassumed mean 𝐴 = 22000
Unit-3 FREQUAENCY DISTRIBUTION
RAI UNIVERSITY, AHMEDABAD 15
Employees Income(Rs.) ( 𝑿 − 𝟐𝟐𝟎𝟎𝟎) = 𝒅
1 14,780 -7220
2 15,760 -6240
3 26,690 +4690
4 27,750 +5750
5 24,840 +2840
6 24,920 +2920
7 16,100 -5900
8 17,810 -4190
9 27,050 +5050
10 26,950 +4950
N=10 ∑ 𝑑 = 2650
 𝑋̅ = 𝐴 +
∑ 𝑑
𝑁
 𝐻𝑒𝑟𝑒 𝐴 = 22000,∑ 𝑑 = 2650, 𝑁 = 10
 𝑋̅ = 22000 +
2650
10
= 22,265
 Hence the average income is Rs. 22,265.
4.3 Calculationof Arithmetic Mean-DiscretefrequencyDistribution:
In discrete series arithmetic mean may be computed by applying
1. Direct Method
2. Short-Cut Method
4.3.1DirectMethod:
The formula for computing mean is
Unit-3 FREQUAENCY DISTRIBUTION
RAI UNIVERSITY, AHMEDABAD 16
𝑋̅ =
∑ 𝑓𝑥
𝑁
Where 𝑓 = frequency
𝑋 = The variable in Question
𝑁 = Total number of Observations i.e. ∑ 𝑓
Steps for finding Arithmetic Mean for Discrete frequency Distribution:
1. Multiply the frequency of each row with the variable and obtain the
total ∑ 𝑓𝑋
2. Divide the total obtained by step(i) by the number of observations ,
i.e. Total frequency
Example-- From the following data of the marks obtained by 60 students of a
class calculatesthe arithmetic mean by Direct Method:
Marks No. of Students
20 8
30 12
40 20
50 10
60 6
70 4
Solution:
Let the marks be denoted by 𝑋 and the number of students by 𝑓.
Calculation of Arithmetic Mean
Marks(𝑿) No. of
Students(𝒇)
𝒇𝑿
20 8 160
Unit-3 FREQUAENCY DISTRIBUTION
RAI UNIVERSITY, AHMEDABAD 17
30 12 360
40 20 800
50 10 500
60 6 360
70 4 280
𝑵 = 𝟔𝟎 ∑𝒇𝒙 = 𝟐𝟒𝟔𝟎
 𝑋̅ =
∑ 𝑓𝑋
𝑁
=
2460
60
= 41
Hence, the average marks=41
4.3.2 Short-Cut Method:
According to this method, 𝑋̅ = 𝐴 +
∑ 𝑓𝑑
𝑁
Where 𝐴 = Assumed mean ;
𝑑 = ( 𝑋 − 𝐴);
𝑁 = Total number of observations i.e., ∑ 𝑓.
Steps for finding Arithmetic Mean for Discrete frequency Distribution:
1. Take an assumed mean.
2. Take the deviations of the variable X from the assumed mean and
denote the deviations by 𝑑.
3. Multiply these deviations with the respective frequency and take the
total ∑ 𝑓𝑑.
4. Divide the total obtained in third step by the total frequency.
Example--From the following data of the marks obtained by 60 students of a
class, calculatethe arithmetic mean by Shot-cut Method:
Marks No. of Students
20 8
Unit-3 FREQUAENCY DISTRIBUTION
RAI UNIVERSITY, AHMEDABAD 18
30 12
40 20
50 10
60 6
70 4
Solution:
Supposeassumed mean A= 40
Calculation of Arithmetic Mean by short-cut Method
Marks(𝑿) No. of
Students(𝒇)
( 𝑿 − 𝟒𝟎) = 𝒅 𝒇𝒅
20 8 -20 -160
30 12 -10 -120
40 20 0 0
50 10 +10 +100
60 6 +20 +120
70 4 +30 +120
𝑁 = 60 ∑ 𝑓𝑑 = 60
 𝑋̅ =
∑ 𝑓𝑑
𝑁
= 40 +
60
60
= 40 + 1 = 41
Hence the Arithmetic mean by Shortcut method is =60
 Note:We can Observe that value of Arithmetic mean does not change in
both the method . so, we can use any one for finding arithmetic mean.
4.4 Calculationof Arithmetic Mean – Continuous Frequency Distribution
Unit-3 FREQUAENCY DISTRIBUTION
RAI UNIVERSITY, AHMEDABAD 19
In continuous frequency distribution arithmetic mean may be computed by
applying any of the following methods:
1. Direct Method
2. Short-Cut Method
4.4.1 DirectMethod: (Meanof Continuous frequency distribution)
When direct method is used
𝑿̅ =
∑ 𝒇𝒎
𝑵
Where, m = mid-point of various classes
𝑓 = Frequency of each class
𝑁 = The total frequency
Steps for finding Arithmetic mean by DirectMethod (C.F.D):
1. Obtain the mid-point of each class and denote it by m.
2. Multiply these midpoints by the respective frequency of each class
and obtain the total ∑ 𝑓𝑚
3. Divide the total obtained in step(i) by the sum of the frequency,i.e.,N.
Example--.from the following data compute arithmetic mean by direct
method:
Marks 0-10 10-20 20-30 30-40 40-50 50-60
No. of Students 5 10 25 30 20 10
Solution:
Calculation for Arithmetic Mean
Marks(𝑿) Mid-points
(𝒎)
No. of Students
(𝒇)
𝒇𝒎
0-10 5 5 25
10-20 15 10 150
Unit-3 FREQUAENCY DISTRIBUTION
RAI UNIVERSITY, AHMEDABAD 20
20-30 25 25 625
30-40 35 30 1050
40-50 45 20 900
50-60 55 10 550
𝑵 = 𝟏𝟎𝟎 ∑ 𝒇𝒎 = 𝟑, 𝟑𝟎𝟎
 𝑋̅ =
∑ 𝑓𝑚
𝑁
=
3300
100
= 33
Hence The value of Arithmetic mean is 33.
4.4.2 Short-Cut Method: (Meanof Continuous frequency distribution)
When short-cut method is used, arithmetic mean is computed by applying the
following formula:
𝑋̅ = 𝐴 +
∑ 𝑓𝑑
𝑁
Where A= Assumed mean
𝑑 = Deviations of mid-points from assumed mean i.e.,(m-A)
𝑁 = Total number of observations
Steps for finding Arithmetic Mean by Short-Cut Method: (C.F.D)
1. Take an assumed mean
2. From the mid-point of each class deductthe assumed mean.
3. Multiply the respective frequencies of each class by these deviations
and obtain the total ∑ 𝑓𝑑.
4. Apply the formula 𝑋̅ = 𝐴 +
∑ 𝑓𝑑
𝑁
Example --from the following data compute arithmetic mean by Short-Cut
method:
Marks 0-10 10-20 20-30 30-40 40-50 50-60
No. of Students 5 10 25 30 20 10
Unit-3 FREQUAENCY DISTRIBUTION
RAI UNIVERSITY, AHMEDABAD 21
Solution:
Assumed that Arithmetic Mean =35
N= 100
Calculation of Arithmetic mean by short-cut method
Marks(𝑿) Mid-points
(𝒎)
No. of Students
(𝒇)
( 𝒎 − 𝟑𝟓)
= 𝒅
𝒇𝒅
0-10 5 5 -30 -150
10-20 15 10 -20 -200
20-30 25 25 -10 -250
30-40 35 30 0 0
40-50 45 20 +10 +200
50-60 55 10 +20 +200
𝑵 = 𝟏𝟎𝟎 ∑ 𝒇𝒅 = −𝟐𝟎𝟎
 𝑋̅ = 𝐴 +
∑ 𝑓𝑑
𝑁
= 35 −
200
100
= 35 − 2 = 33
Hence the value of Arithmetic mean by Short-Cut method for continuous
frequency distribution is 33.
Example-- The mean marks of 100 students were found to be 40. Later on it
was discoveredthat a score of 53 was misread as 83.Findthe correctmean
corresponding to correctScore.
Solution:
 We are given 𝑁 = 100, 𝑋̅ = 40
 Since 𝑋̅ =
∑ 𝑋
𝑁
 ∑ 𝑋 = 𝑁𝑋̅ = 100× 40 = 4000
 But this is not correct ∑ 𝑋
Unit-3 FREQUAENCY DISTRIBUTION
RAI UNIVERSITY, AHMEDABAD 22
 Correct ∑ 𝑋 = Incorrect ∑ 𝑋 − Wrong item + Correct item
 Correct ∑ 𝑋 = 4000− 83 + 53 = 3970
 ∴ Correct 𝑋̅ =
correct ∑ 𝑋
N
=
3970
100
= 39.7
Hence the Correct mean=39.7
4.5 Weightedarithmetic mean
One of the limitations of the arithmetic mean discussed above is that it gives equal
importance to all the items. But there are cases where the relative importance of the
different items.The formula for computing weighted arithmetic mean is:
𝑋 𝑤
̅̅̅̅ =
∑ 𝑊𝑋
∑ 𝑊
 Where 𝑋 𝑤
̅̅̅̅ represents the weighted arithmetic mean; X represents the
variable values.
W represents the weights attached to variable values.
Steps for finding Weighted mean:
1. Multiply the weights by the variable X and obtain the total ∑ 𝑊𝑋
2. Divide this total by the sum of the weights, i.e.,∑ 𝑊.
Example-- Calculate weightedaverage ofthe following data:
Course BA BSc MA MCA MBA
%of Pass 70 65 75 90 99
No of
Students
20 30 30 50 40
Solution:
 Weighted Average =
∑ 𝑊𝑋
∑ 𝑊
 Weighted Average =13300 / 170 = 76.47
Calculation for weighted Average (Mean)
Unit-3 FREQUAENCY DISTRIBUTION
RAI UNIVERSITY, AHMEDABAD 23
% of Pass X No of Student W XW
70 20 1400
65 30 1950
75 30 2250
90 50 4500
80 40 3200
X=380 W=170 13300
Example--A train runs 25 miles at a speedof 30 m.p.h., another50 miles at a
speedof 40 m.p.h, then due to repairs a track travels for 6 minutes at a speed
of 10 m.p.h. what is the average speedin miles per hour?
Solution:
Time taken in covering 25 miles at a speed of 30 m.p.h=50 minutes. Time taken in
covering 50 miles at a speed of 40 m.p.h = 75 minutes. Distance covered in 6
minutes at a speed of 10 m.p.h = 1 mile. Time taken in covering 24 miles at a
speed of 24 m.p.h = 60 minute Therefore, taking the time taken as weights we have
the weighted mean as
Speed (in m.p.h)
(𝑿)
Time taken
(in min) W
𝑾𝑿
30 50 1500
40 75 3000
10 6 60
24 60 1440
∑ 𝑊 = 191 ∑ 𝑊 𝑋 = 6000
 ∴ Weighted arithmetic mean =𝑋 𝑤
̅̅̅̅ =
∑ 𝑊𝑋
∑ 𝑊
=
6000
191
= 31.41 𝑚. 𝑝. ℎ.
Unit-3 FREQUAENCY DISTRIBUTION
RAI UNIVERSITY, AHMEDABAD 24
Hence value of weighted arithmetic mean is 31.41 𝑚. 𝑝. ℎ.
5.1 MathematicalProperties Of Arithmetic Mean:
The following are a few important mathematical properties of the arithmetic mean:
1. The sum of the deviations of the items from the arithmetic mean (taking
signs into account) is always zero. i.e. ∑(𝑋 − 𝑋̅) = 0
2. The sum of the squared deviations of the items from arithmetic mean is
minimum, that is, less than the sum of the squared deviations of the items
from any other value.
3. It is clear that ∑(𝑋 − 𝑋̅)2
is greater. This Property that the sum of the
squares of items is least from the mean is of immense use regression
analysis which shall be discussed later.
5.2 Merits of arithmetic mean:
Arithmetic mean is the simplest measurement of central tendency of a group. It is
extensively used because:
 It is easy to calculate and easy to understand.
 It is based on all the observations.
 It is rigidly defined.
 It provides good basis of comparison.
 It can be used for further analysis and algebraic treatment.
5.3 Demerits of the arithmetic mean:
 It is affected by the extreme values.
 It may lead to a wrong conclusion.
 It is unrealistic.
 Arithmetic mean cannot be obtained even if single observation is missing
 It cannot be identified observation or graphic method
6.1Median:
Unit-3 FREQUAENCY DISTRIBUTION
RAI UNIVERSITY, AHMEDABAD 25
The median is the middle score for a set of data that has been arranged in order of
magnitude.
If the number of events are even then the average of two middle are taken.
The median is better for describing the typical value.
Example:
In order to calculate the median, supposewe have the data below:
65 55 89 56 35 14 56 55 87 45 92
We first need to rearrange that data into order of magnitude (smallest first):
14 35 45 55 55 56 56 65 87 89 92
Our median mark is the middle mark - in this case, 56 (highlighted in bold).
The median by definition refers to the middle value in a distribution. The median is
that value of the series which divides the group into two equal parts, one part
comprising all values greater than the median value and the other part comprising
all the values smaller than the median value.
6.2 Steps for Calculationof Median – Individual Observations
1. Arrange the data in ascending or descending order of magnitude.
(Both arrangements would give the same answer)
2. Apply the formula Median = 𝑀 =
𝑁+1
2
Example-- Find the median for the following data:
5, 15, 10, 15, 5, 10, 10, 20, 25 and 15.
Solution:
First of all we have to arrange all the Observations in ascending order
5, 5, 10, 10, 10, 15, 15, 15, 20, 25
Here by Observation we can say that 𝑁 = 10
𝐻𝑒𝑛𝑐𝑒, Median 𝑀 =
𝑁+1
2
=
10+1
2
= 5.5th item =
10+15
2
= 12.5
Example-- Find the median for the following data:
Unit-3 FREQUAENCY DISTRIBUTION
RAI UNIVERSITY, AHMEDABAD 26
25900,26950,27020,27200,28280
Solution:
First of all we have to arrange all the Observations in ascending order
25900, 26950, 27020, 27200, 28280
Here by Observation we can say that 𝑁 = 5
𝐻𝑒𝑛𝑐𝑒, Median 𝑀 =
𝑁+1
2
=
5+1
2
=
6
2
= 3𝑟𝑑 item =27020
6.3 Steps for Calculationof median of continuous frequency distribution:
1. Determine the particular class in which the value of median lies.
2. Use
𝑁
2
as the rank of the median and not
𝑁+1
2
.
3. After ascertaining the class in which median lies, the following formula is
used for determining the exact value of median
𝑀𝑒𝑑𝑖𝑎𝑛 = 𝐿 +
𝑁
2
− 𝑐. 𝑓.
𝑓
× 𝑖
Where,
𝐿 = Lower limit of the median class i.e., the class in which the middle item of
the distribution lies.
𝑐. 𝑓. = Cumulative frequency of the class preceding the median class or sum of the
Frequency of the frequencies of all classes lower than the median class.
𝑓 = Simple frequency of the Median class.
𝑖 = The class interval of the Median class.
Example--: Find the median of the following data.
Cost 10-20 20-30 30-40 40-50 50-60
Items in a
group
4 5 3 6 3
Solution:
Unit-3 FREQUAENCY DISTRIBUTION
RAI UNIVERSITY, AHMEDABAD 27
Calculation for Median
Cost Number of items in the group Cumulative frequency
10-20 4 4
20-30 5 9
30-40 3 12
40-50 6 18
50-60 3 21
 Here N=21 ⇒
𝑁
2
= 10.5
 The median class is 30-40.
From Formula,
𝑀𝑒𝑑𝑖𝑎𝑛 = 𝐿 + (
𝑁
2
− 𝑐𝑓
𝑓
) × 𝑖
 Here, L=30, 𝑖 = 10, 𝑐𝑓 = 9
 𝑀𝑒𝑑𝑖𝑎𝑛 = 30+
(10.5−9)
12
× 10 = 30 + 1.25 = 31.25
7.1 MathematicalProperty of Median:
1. The sum of the deviations of the items from median, ignoring signs, is the
least.
For example, the median of 4,6,8,10,12 is 8. The deviations from 8 ignoring signs
are 4, 2, 0, 2, 4 and the total is 12.This total is smaller than the one obtained if
deviations are taken from any other value. Thus if deviations are taken from 7,
values ignoring signs would be 3, 1, 1, 3, 5 and the total 13.
7.2 Merits of Median:
 It is easy to calculate and easy to understand.
 It is based on all the observations.
 It is rigidly defined.
Unit-3 FREQUAENCY DISTRIBUTION
RAI UNIVERSITY, AHMEDABAD 28
 It eliminates the impact of extreme values.
 It can be used for further analysis and algebraic treatment.
 Median can be found out just by inspection in some cases.
7.3Demerits Of Median:
 It simply ignores the extreme values.
 It may lead to a wrong conclusion. When distribution of observations is
Irregular.
 The median is estimated in continuous case.
8.1 Mode:
The value of the variable which occurs mostfrequently in a distribution is called
the mode. Mode = 3 Median – 2 Mean
8.2 Calculationof Mode – Individual Operations.
For determining mode count the number of times the various values repeat
themselves and the value occurring maximum number of times is the mdal value
.The more often the modal value appears relatively,the more valuable the measure
is an average to represent data.
Example--The following is the number of problems that Ms. Matty assigned
for homework on 10 different days. What is the mode?
8, 11, 9, 14, 9, 15, 18, 6, 9, 10
Solution:
Ordering the data from least to greatest, we get:
6, 8, 9, 9, 9, 10, 11, 14, 15, 18
The scorewhich occurs mostoften is 9.
Therefore, the mode is 9.
Unit-3 FREQUAENCY DISTRIBUTION
RAI UNIVERSITY, AHMEDABAD 29
Example-2 In a crashtest, 11 cars were testedto determine what impact speed
was required to obtain minimal bumper damage. Find the mode of the speeds
given in miles per hour below.
24, 15, 18, 20, 18, 22, 24, 26, 18, 26, 24
Solution:
Ordering the data from least to greatest, we get:
15, 18, 18, 18, 20, 22, 24, 24, 24, 26, 26
Since both 18 and 24 occurthree times, the modes are 18 and 24 miles per hour.
This data set is bimodal.
Example-3 A marathon race was completed by 5 participants. What is the
mode of these times given in hours?
2.7 hr, 8.3 hr, 3.5 hr, 5.1 hr, 4.9 hr
Solution:
Ordering the data from least to greatest, we get:
2.7, 3.5, 4.9, 5.1, 8.3
Since each value occurs only once in the data set, there is no mode for this set of
data.
8.2Steps for Calculationof Mode For Continuous frequency Distribution:
1. Constructthe table
Unit-3 FREQUAENCY DISTRIBUTION
RAI UNIVERSITY, AHMEDABAD 30
2. Find the Modal class
3. Find out Mode class by using N / 2
4. Apply the formula
𝑀𝑜𝑑𝑒 = 𝐿 +
(𝑓1 − 𝑓0)
2𝑓1 − 𝑓0 − 𝑓2
× 𝑖
Where, L = Lower limit of the Modal class
𝑓1Frequency of the Modal class
𝑓0 = Frequency of the class preceding the Modal class
𝑓2 =Frequency of the class succeeding Modal class
i = Class interval of modal class
Example--: Calculate mode of the following data:
Marks 10-20 20-30 30-40 40-50 50-60
F 5 20 25 15 5
Solution:
Construction of the table to find Mode:
Marks F
10-20 5
20-30 20
30-40 25
40-50 15
50-60 5
Modal class is 30-40 since highest frequency occurs here i.e. frequency of that
class is =25
𝑀𝑜𝑑𝑒 = 𝐿 +
(𝑓1 − 𝑓0)
2𝑓1 − 𝑓0 − 𝑓2
× 𝑖
Where, L = Lower limit of the Modal class = 30
Unit-3 FREQUAENCY DISTRIBUTION
RAI UNIVERSITY, AHMEDABAD 31
𝑓1 = Frequency of the Modal class = 25
𝑓0 = Frequency of the class preceding the Modal class = 20
𝑓2 =Frequency of the class succeeding Modal class =15
i = Class interval of modal class =10
 𝑀𝑜𝑑𝑒 = 30+
(25−20)
(2×25)−20−15
× 10
 𝑀𝑜𝑑𝑒 = 30+
50
15
 𝑀𝑜𝑑𝑒 = 33.33
9.1Merits of Mode:
 It is easy to calculate and easy to understand.
 It eliminates the impact of extreme values
 It can be identified by using graphical method
9.2Demerits of Mode
 It is not suitable for further mathematical treatments.
 It may lead to a wrong conclusion. When bimodal distribution.
 It is difficult to compute in some cases.
 Mode is influenced by length of the class interval.
10.1 Introduction
Averages give us information of concentration of the observations about the central
part of the distribution. But they fail to give anything further about the data.
According to George Simpson and Fritz Kafka, “An average does not tell the full
Story. It is hardly fully representative of a mass, unless we know the manner in
which the individual items scatter around it.
“Dispersion is the measure of the variations of the items.’’- A.L Bowley
10.2 RANGE
The range is the difference between two extreme values of the given observations
Range = Largest value – Smallest value
Unit-3 FREQUAENCY DISTRIBUTION
RAI UNIVERSITY, AHMEDABAD 32
10.3 Co-efficientof Range
Co- efficient of Range =
𝐿𝑎𝑟𝑔𝑒𝑠𝑡 𝑣𝑎𝑙𝑢𝑒−𝑆𝑚𝑎𝑙𝑙𝑒𝑠𝑡 𝑣𝑎𝑙𝑢𝑒
𝐿𝑎𝑟𝑔𝑒𝑠𝑡 𝑣𝑎𝑙𝑢𝑒+𝑆𝑚𝑎𝑙𝑙𝑒𝑠𝑡 𝑣𝑎𝑙𝑢𝑒
=
𝐿−𝑆
𝐿+𝑆
Example-- Find the Co-efficient of range of Marks of 10 students from the
following
65,35,48,99,56,88,78,20,66,53
Solution:
 Range = L – S
 Range = 99 -20
 Range = 79
 Co- efficient of Range =
99−20
99+20
=
79
109
= 0.72
10.4 Merits:
1. It is easy to compute and understand.
2. It gives an idea about the distribution immediately.
10.5 Demerits:
1. Calculation range depends only on the basis of extreme items, hence it is
not reliable.
2. It is not applied to open end cases
3. Not suitable for mathematical treatments.
11.1Quartile Deviation:
The range which includes the middle 50 per cent of the distribution.That is one
quarter of the observations at the lower end, another quarter of the observations at
the upper end of the distribution are excluded in computing the interquartile
range.In the other words, interquartile range represents the difference between the
third quartile and the first quartile.
Symbolically,
Unit-3 FREQUAENCY DISTRIBUTION
RAI UNIVERSITY, AHMEDABAD 33
𝐼𝑛𝑡𝑒𝑟𝑞𝑢𝑎𝑟𝑡𝑖𝑙𝑒 𝑟𝑎𝑛𝑔𝑒 = 𝑄3 − 𝑄1
Very often the interquartile range is reduced to the form of the semi-interquartile
range or quartile deviation by dividing it by 2.
𝑄𝑢𝑎𝑟𝑡𝑖𝑙𝑒 𝐷𝑒𝑣𝑖𝑎𝑡𝑖𝑜𝑛 𝑜𝑟 𝑄. 𝐷. =
𝑄3 − 𝑄1
2
11.2 Coefficientof Quartile Deviation:
Quartile deviation is an absolute measure of dispersion .The relative measure
corresponding to this measure, called the coefficient of quartile deviation, is
calculated as follows.
𝐶𝑜𝑒𝑓𝑓𝑖𝑐𝑖𝑒𝑛𝑡 𝑜𝑓 𝑄. 𝐷. =
𝑄3 − 𝑄1
2
𝑄3 + 𝑄1
2
=
𝑄3 − 𝑄1
𝑄3 + 𝑄1
Example--find out the value of quartile deviation and its coefficientfrom the
following data:
Roll No. 1 2 3 4 5 6 7
Marks 20 28 40 12 30 15 50
Solution:
 Marks arranged in ascending order: 12 15 20 28 30 40 50
 𝑄1 = Size of
𝑁+1
4
th item =
7+1
4
= 2𝑛𝑑 item
 Size of 2nd item is 15. Thus 𝑄1 = 15
 𝑄3 = size of 3 (
𝑁+1
4
) 𝑡ℎ item = Size of (
3×8
4
) 𝑡ℎ item =6th item
 Size of 6th item is 40.Thus 𝑄3 = 40
𝑄. 𝐷. =
𝑄3 − 𝑄1
2
=
40 − 15
2
= 12.
 Now we have to find coefficient of Q.D.
𝐶𝑜𝑒𝑓𝑓𝑖𝑐𝑖𝑒𝑛𝑡 𝑜𝑓 𝑄. 𝐷. =
𝑄3−𝑄1
𝑄3+𝑄1
=
40−15
40+15
=
25
55
= 0.455
Unit-3 FREQUAENCY DISTRIBUTION
RAI UNIVERSITY, AHMEDABAD 34
Example--Compute coefficientnofnquartile deviation from the following data:
Marks 10 20 30 40 50 60
No. of Students 4 7 15 8 7 2
Solution:
Calculation of coefficient of quartile deviation
Marks Frequency c.f.
10 4 4
20 7 11
30 15 26
40 8 34
50 7 41
60 2 43
 𝑄1 = size of
𝑁+1
4
𝑡ℎ item =
43+1
4
= 11th item
 Size of 11th item is 20. Thus 𝑄1 = 20
 𝑄3 = Size of 3 (
𝑁+1
4
) 𝑡ℎ item =
3×44
4
= 33rd item
 Size of 33rd item is 40. Thus, 𝑄3 = 40
𝑄. 𝐷. =
𝑄3−𝑄1
2
=
40−20
2
= 10
 𝐶𝑜𝑒𝑓𝑓𝑖𝑐𝑖𝑒𝑛𝑡 𝑜𝑓 𝑄. 𝐷. =
𝑄3−𝑄1
𝑄3+𝑄1
=
40−20
40+20
= 0.333
12.1MeanDeviation:
Mean deviation is the arithmetic mean of the difference of a series computed from
any measure of central tendency i.e., Deviations from Mean or Mode or Median.
All the deviation’s absolute values are considered.
The mean deviation is also known as the average deviation. It is the average
difference between the items in a distribution and the median or mean of that
Unit-3 FREQUAENCY DISTRIBUTION
RAI UNIVERSITY, AHMEDABAD 35
series. Theoritically there is an advantage in taking the deviations from median
because the sum of deviations of items from median is minimum when signs are
ignored.
12.2Computation of MeanDeviation-Individual Observations:
If 𝑋1, 𝑋2, 𝑋3, …… 𝑋 𝑁 are 𝑁 given observations then the deviation about an average
𝐴 is given by
𝑀. 𝐷. =
1
𝑁
∑| 𝑋 − 𝐴| =
1
𝑁
∑| 𝐷| 𝑜𝑟
∑| 𝐷|
𝑁
Where | 𝐷| = | 𝑋 − 𝐴|.
Read as mod (X-A) is the modulus value or absolute value of the deviation
ignoring plus and minus signs.
12.2 Steps for Computation of mean deviation: (Indiviadual Observations)
1. Compute the median of the series.
2. The deviations of items from median ignoring ± signs and denote these
deviations by | 𝐷|.
3. Obtain the total of these deviations,ie. ∑| 𝐷|.
4. Divide the total obntained in step (3) by the total number of observations.
12.3Coefficientof MeanDeviation:
The relative measure corresponding to the mean deviation called the coefficient of
mean deviation is obtained by dividing mean deviation by the particular average
used in computing mean deviation. Thus if mean deviation has been computed
from median, the coefficient of mean deviation shall be obtained by dividing mean
deviation by mean, median or mode.
𝐶𝑜𝑒𝑓𝑓𝑖𝑐𝑖𝑒𝑛𝑡 𝑜𝑓 𝑀. 𝐷. =
𝑀.𝐷.
𝑀𝑒𝑎𝑛 ,𝑀𝑒𝑑𝑖𝑎𝑛 𝑜𝑟 𝑀𝑜𝑑𝑒
Example--Calculate mean deviation and coefficient of mean deviation from
the following data:
Unit-3 FREQUAENCY DISTRIBUTION
RAI UNIVERSITY, AHMEDABAD 36
100 200 300 400 500 600 700
Solution:
Calculation for Mean Deviation
𝑿 | 𝑫| = | 𝑿 − 𝑨|=| 𝑿 − 𝟒𝟎𝟎|
100 300
200 200
300 100
400 0
500 100
600 200
700 300
∑| 𝑫| = 𝟏𝟐𝟎𝟎
 Arithmetic Mean=A =
∑ 𝑋𝑖
𝑁
=
2800
7
= 400
 Mean Deviation= 𝑀. 𝐷. =
1
𝑁
∑| 𝑋 − 𝐴| =
1
𝑁
∑| 𝐷|
𝑀. 𝐷. =
1200
7
= 171.42
 𝐶𝑜𝑒𝑓𝑓𝑖𝑐𝑖𝑒𝑛𝑡 𝑜𝑓 𝑀. 𝐷. =
𝑀.𝐷.
𝑀𝑒𝑎𝑛
=
171.42
400
= 0.4285
 𝐶𝑜𝑒𝑓𝑓𝑖𝑐𝑖𝑒𝑛𝑡 𝑜𝑓 𝑀. 𝐷 = 0.43
12.2.2 Calculationof MeanDeviation-(Discrete frequency distribution):
In discrete series the formula for calculating mean deviation is
𝑀. 𝐷. =
∑ 𝑓| 𝐷|
𝑁
Where | 𝐷| denote deviation from median ignoring signs.
Unit-3 FREQUAENCY DISTRIBUTION
RAI UNIVERSITY, AHMEDABAD 37
Steps for calculation:
1. Calculate the median of the series.
2. Take the deviations of the items from median ignoring signs and denote
them by | 𝐷|.
3. Divide the total obtained in step (ii) by the number of observations.This
gives us the value of mean deviation.
Example--Claculate Meandeviation from the following series
X 10 11 12 13 14
f 3 12 18 12 3
Solution:
Calculation of Mean Deviation
X 𝒇 | 𝑫| 𝒇| 𝑫| 𝒄. 𝒇.
10 3 2 6 3
11 12 1 12 15
12 18 0 0 33
13 12 1 12 45
14 3 2 6 48
𝑁 = 48 ∑ 𝑓| 𝐷|=36
 𝑀. 𝐷.=
∑ 𝑓| 𝐷|
𝑁
 Median = Size of
𝑁+1
2
𝑡ℎ item =
48+1
2
= 24.5𝑡ℎ item
 Size of 24.5th item is 12, hence Median = 12
 𝑀. 𝐷. =
∑ 𝑓| 𝐷|
𝑁
=
36
48
= 0.75
12.2.3 Calculationof MeanDeviation– Continuous Frequency distribution
Unit-3 FREQUAENCY DISTRIBUTION
RAI UNIVERSITY, AHMEDABAD 38
For calculating mean deviation in continuous series the procedureremains the
same as discussed above.Theonly difference is that here we have to obtain the mid
point of the various classes and take deviations of these points from median. The
formula is same, i.e.,
𝑀. 𝐷. =
∑ 𝑓| 𝐷|
𝑁
Example-- Calculate the mean deviation from mean for the following data:
Class Interval 2-4 4-6 6-8 8-10
frequency 6 8 4 2
Solution:
Calculation for Mean Deviation
Class Mid
value(m)
Frequency 𝒇𝒎 | 𝑫|
= | 𝑿 − 𝑨|
𝒇| 𝑫|
2-4 3 6 18 2.2 13.2
4-6 5 8 40 0.2 1.6
6-8 7 4 28 1.8 7.2
8-10 9 2 18 3.8 7.6
Total 20 104 27.6
 𝐴 =
∑ 𝑓𝑚
∑ 𝑓
=
104
20
= 5.2
 𝑀. 𝐷. =
∑ 𝑓| 𝐷|
𝑁
=
27.6
20
= 1.48 (ℎ𝑒𝑟𝑒 𝑁 = ∑ 𝑓 = 20)
 𝑀. 𝐷. = 1.48
12.4Merits of MeanDeviation:
1. It is simple to understand and easy to calculate
2. The computation process is based on all items of the series
Unit-3 FREQUAENCY DISTRIBUTION
RAI UNIVERSITY, AHMEDABAD 39
3. It is less affected by the extreme items.
4. This measure is flexible, Since it can be calculated from mean, meadian, or
mode.
5. This measure is rigidly defined.
12.5Demerits of Mean Deviation:
1. This measure is not a very accurate measure of dispersion.
2. Not suitable for further mathematical calculation.
3. It is rarely used.
4. Absolute values are considered, mathematically unsound and illogical.
13.1 Standard Deviation:
The famous statistician karl pearsonintroduced the conceptof standard deviation
in 18
This is the most accepted measure of dispersion and also widely used in many
statistical applications. Standard deviation is also referred as root-mean square
deviation or Mean square error. It gives accurate results.
The standard deviation is also denoted by the greek letter ( 𝜎).
13.2Variance:
The term variance was used to describe the square of standard deviation by
R.A.Fisher in 1913.
The conceptof variance is highly important in advanced work where it is possible
to split the total into several parts,each attributable to one of the factors causing
variation in their original series.
Variance is defined as follows
𝑉𝑎𝑟𝑖𝑎𝑛𝑐𝑒 =
∑( 𝑋−𝐴)2
𝑁
Thus variance is nothing but the square of the standard deviation
𝑖. 𝑒., 𝑉𝑎𝑟𝑖𝑎𝑛𝑐𝑒 = 𝜎2
𝑜𝑟 𝜎 = √ 𝑉𝑎𝑟𝑖𝑎𝑛𝑐𝑒
Unit-3 FREQUAENCY DISTRIBUTION
RAI UNIVERSITY, AHMEDABAD 40
In a frequency distribution where deviations are taken from assumed mean
variance may directly be computed as follows
𝑉𝑎𝑟𝑖𝑎𝑛𝑐𝑒 = {
∑ 𝑓𝑑2
𝑁
− (
∑ 𝑓𝑑
𝑁
)
2
} × 𝑖2
𝑤ℎ𝑒𝑟𝑒, 𝑑 =
𝑋−𝐴
𝑖
and 𝑖=class interval
13.2. CalculationOfStandard Deviation- Individual Observation
There are two method of calculating standard deviation in an individual
observation:
(i) Direct Method – Deviation taken from actual mean
(ii) Short- cut Method – Deviation taken from assumed mean
13.2.1 (i) DirectMethod:
The following are the steps:
1. Find out actual mean of the given observations.
2. Compute deviation of each observation from the mean (𝑋 − 𝑀𝑒𝑎𝑛).
3. Square the deviations and find out the sum i.e. ( 𝑋 − 𝐴)2
4. Divide the total by the number of observations and take square rootof the
quotient,the value is standard deviation.
𝜎 = √
∑( 𝑋 − 𝐴)2
𝑁
Example-- Calculate the standard deviation from the following data:
𝟏𝟓, 𝟏𝟐, 𝟏𝟕, 𝟏𝟎, 𝟐𝟏, 𝟏𝟖, 𝟏𝟏, 𝟏𝟔
Solution:
Calculation of S.D. from Mean
Values (𝑿) (𝑿 − 𝑨) ( 𝑿 − 𝑨) 𝟐
Unit-3 FREQUAENCY DISTRIBUTION
RAI UNIVERSITY, AHMEDABAD 41
15 0 0
12 -3 9
17 2 4
10 -5 25
21 6 36
18 3 9
11 -4 16
16 1 1
𝑿 = 𝟏𝟐𝟎 ( 𝑿 − 𝑨) 𝟐
= 𝟏𝟎𝟎
 𝐴 =
∑ 𝑋𝑖
𝑁
=
120
8
= 15
 S.D.=𝜎 = √
∑( 𝑋−𝐴)2
𝑁
= √
100
8
 𝜎 = 3.53
 𝑣𝑎𝑟𝑖𝑎𝑛𝑐𝑒 = 𝜎2
= 12.46
13.2.2(ii) Short-Cut Method - Deviation takenfrom assumedmean
This method is used when arithmetic mean is fractional value. A deviation from
fractional value leads to tedious task. To save calculation time,we apply this
method the formula is
𝜎 = √
∑ 𝑑2
𝑁
− (
∑ 𝑑
𝑁
)
2
Where, d = deviations from assumed mean =(𝑋 − 𝐴)
𝑁 = Number of Observations
Steps for calculations:
1. Take the deviations of the items from an assumed mean, i.e. obtain
( 𝑋 − 𝐴). Denote these deviations by d.
2. Take the total of these deviations. i.e., obtain ∑ 𝑑.
Unit-3 FREQUAENCY DISTRIBUTION
RAI UNIVERSITY, AHMEDABAD 42
3. Square these deviations and obtain the total ∑ 𝑑2
.
4. Substitute the values of ∑ 𝑑2
,∑ 𝑑 𝑎𝑛𝑑 𝑁 in the above formula.
Example--Bloodserum cholesterollevels of10 persons are as under:
240, 260, 290,245, 255, 288,272, 263, 277,251
Calculate standard deviation and variance with the help of assumed
mean.
Solution:
Calculation of Standard Deviation
𝑿 𝒅 = (𝑿 − 𝟐𝟔𝟒) 𝒅 𝟐
240 -24 576
260 -4 16
290 +26 676
245 -19 361
255 -9 81
288 +24 576
272 +8 64
263 -1 1
277 +13 169
251 -13 169
∑ 𝑿 =2641 ∑ 𝑑 = +1 ∑ 𝑑2
= 2689
 𝜎 = √
∑ 𝑑2
𝑁
− (
∑ 𝑑
𝑁
)
2
 Now ∑ 𝑑2
= 2689,∑ 𝑑 = +1, 𝑁 = 10
Unit-3 FREQUAENCY DISTRIBUTION
RAI UNIVERSITY, AHMEDABAD 43
 ∴ 𝜎 = √2689
10
− (
1
10
)
2
 ∴ 𝜎 = √268.9− 0.01
 ∴ 𝜎 = 16.398
 𝑉𝑎𝑟𝑖𝑎𝑛𝑐𝑒 = 𝜎2
= (16.398)2
= 268.894
13.3 Calculationof Standard Deviation– Discrete frequency distribution
For calculating standard deviation in discrete series, any of the following methods
may be applied:
1. Actual Mean Method
2. Assumed Mean Method
3. Step deviation method
13.3.1 ActualMeanMethod:
When this method is applied, deviations are taken from the actual mean,The
formula is applied
𝜎 = √
∑ 𝑓( 𝑋 − 𝐴)2
𝑁
However in practice this method is rarely used becauseif the actual mean is in
fraction the calculations take a lot of time.
Steps for calculation:
(1) Compute mean of the observations
(2) Compute deviation from the mean 𝑑 = ( 𝑋 − 𝐴)
(3) Square the deviations d2
and multiply these values with respective frequencies
f i.e., fd2
(4) Sum the products 𝑓𝑑2
and apply the formula
𝜎 = √
∑ 𝑓𝑑2
𝑁
= √
∑ 𝑓( 𝑋 − 𝐴)2
𝑁
Unit-3 FREQUAENCY DISTRIBUTION
RAI UNIVERSITY, AHMEDABAD 44
Example-- Compute standard deviation and variance from the following data
Marks 10 20 30 40 50
Frequency 2 8 10 8 2
Solution:
Construct the table to compute the standard deviation
Marks (X) 𝒇 𝒇𝑿 𝒅 = 𝑿 − 𝟑𝟎 𝒅 𝟐 𝒇𝒅 𝟐
10 2 20 -20 400 800
20 8 160 -10 100 800
30 10 300 0 0 0
40 8 320 10 100 800
50 2 100 20 400 800
𝑓 = 30 𝑓𝑋 = 900 𝑓𝑑2
= 3200
 Mean = 𝐴 =
∑ 𝑋𝑖
𝑁
=
900
30
= 30
 𝜎 = √
∑ 𝑓𝑑2
𝑁
= √
∑ 𝑓( 𝑋−𝐴)2
𝑁
 𝜎 = √
3200
30
 𝜎 = √106.66 = 10.325
 𝑉𝑎𝑟𝑖𝑎𝑛𝑐𝑒 = 𝜎2
= 106.66
13.3.2 AssumedMeanMethod:
When this method is used, the following formula is applied:
𝜎 = √
∑ 𝑓𝑑2
𝑁
− (
∑ 𝑓𝑑
𝑁
)
2
Where 𝑑 = (𝑋 − 𝐴)
Steps for calculation:
1. Assume any one of the given value as assumed mean A
Unit-3 FREQUAENCY DISTRIBUTION
RAI UNIVERSITY, AHMEDABAD 45
2. Compute deviation from the assumed mean (𝑑 = 𝑋 − 𝐴).
3. Multiply these deviations by its frequencies 𝑓𝑑.
4. Square the deviations (𝑑2
) and multiply these values with respective
Frequencies ( 𝑓) i.e., 𝑓𝑑2
5. Sum the products 𝑓𝑑2
and apply the formula 𝜎 = √
∑ 𝑓𝑑2
𝑁
− (
∑ 𝑓𝑑
𝑁
)
2
Example-- Compute standard deviation and variance from the following data
Marks 10 20 30 40 50
Frequency 2 8 10 8 2
Solution: Constructthe table to compute the standard deviation
Marks (X) 𝒇 𝒅 = 𝑿 − 𝟐𝟎 𝒅 𝟐 𝒇𝒅 𝒇𝒅 𝟐
10 2 -10 100 -20 200
20 8 0 0 0 0
30 10 10 100 100 1000
40 8 20 400 160 3200
50 2 30 900 60 1800
𝑓 = 30 𝑓𝑑 = 300 𝑓𝑑2
= 6200
 𝜎 = √
∑ 𝑓𝑑2
𝑁
− (
∑ 𝑓𝑑
𝑁
)
2
 𝑁𝑜𝑤, here ∑ 𝑓𝑑2
= 6200,∑ 𝑓𝑑 = 300, 𝑁 = ∑ 𝑓 = 30
 𝜎 = √6200
30
− (
300
30
)
2
 𝜎 = √206.66 − 100 = √106.66
 𝜎 = 10.325
Unit-3 FREQUAENCY DISTRIBUTION
RAI UNIVERSITY, AHMEDABAD 46
 𝑉𝑎𝑟𝑖𝑎𝑛𝑐𝑒 = 𝜎2
= 106.66
13.3.3StepdeviationMethod:
When this method is used we take deviations of midpoints from an assumed mean
and divide these deviations by the width of class interval,i.e. ′𝑖′
. In case class
intervals are unequal. We divide the deviations of midpoints by the lowest
common factor and use ‘c’ instead of ′𝑖′
in the formula for calculating standard
deviation.
The formula for calculating standard deviation is:
𝜎 = √
∑ 𝑓𝑑2
𝑁
− (
∑ 𝑓𝑑
𝑁
)
2
× 𝑖
Where 𝑑=
𝑋−𝐴
𝑖
and 𝑖 = class interval
Steps for calculation:
Take a common factor and divide that item by all deviations
1. Assume any one of the given value as assumed mean A
2. Compute deviation from the assumed mean (𝑑 =
𝑋−𝐴
𝑖
).
3. Multiply these deviations by its frequencies 𝑓𝑑.
4. Square the deviations (𝑑2
) and multiply these values with respective
Frequencies ( 𝑓) i.e., 𝑓𝑑2
5. Sum the products 𝑓𝑑2
and apply the formula.
Example-- The annual salaries ofa group of employees are given in the
following table:
Salaries 45000 50000 55000 60000 65000 70000 75000 80000
Number of
persons
3 5 8 7 9 7 4 7
Calculate the standard deviation of the salaries.
Unit-3 FREQUAENCY DISTRIBUTION
RAI UNIVERSITY, AHMEDABAD 47
Solution:
Calculation of standard deviation
Salaries (X) No. of
persons(𝒇)
𝒅 =
𝑿 − 𝟔𝟎𝟎𝟎𝟎
𝟓𝟎𝟎𝟎
𝒇𝒅 𝒇𝒅 𝟐
45000 3 -3 -9 27
50000 5 -2 -10 20
55000 8 -1 -8 8
60000 7 0 0 0
65000 9 +1 +9 9
70000 7 +2 +14 28
75000 4 +3 +12 36
80000 7 +4 +28 112
𝑵 = 𝟓𝟎 ∑ 𝒇𝒅 =36 ∑ 𝒇𝒅 𝟐
= 𝟐𝟒𝟎
 𝜎 = √
∑ 𝑓𝑑2
𝑁
− (
∑ 𝑓𝑑
𝑁
)
2
× 𝑖
 Now here ∑ 𝑓𝑑 = 36, ∑ 𝑓𝑑2
= 240 ,𝑁 = 50, 𝑖 = 5000
 ∴ 𝜎 = √240
50
− (
36
50
)
2
× 5000
 ∴ 𝜎 = √4.8− 0.5184× 5000
 ∴ 𝜎 = 10346.01
13.4 Calculationof Standard Deviation– Continuous Series.
In continuous series any of the methods discussed above for discrete frequency
distribution can be used. However, in practice it is the step deviation method that is
most used.
The formula is,
Unit-3 FREQUAENCY DISTRIBUTION
RAI UNIVERSITY, AHMEDABAD 48
𝜎 = √
∑ 𝑓𝑑2
𝑁
− (
∑ 𝑓𝑑
𝑁
)
2
× 𝑖
Where 𝑑 =
𝑚−𝐴
𝑖
, 𝑖 = 𝑐𝑙𝑎𝑠𝑠 𝑖𝑛𝑡𝑒𝑟𝑣𝑎𝑙
Steps for calculation:
1. Find the mid point of various classes.
2. Take the deviations of these mid points from an assumed mean and denote
thrse deviations by 𝑑.
3. Wherever possible take a common factor and denote this column by 𝑑.
4. Multiply the frequencies of each class with these deviations and obtain ∑ 𝑓𝑑.
5. Square the deviations and multiply them with the respective frequencies of
each class and obtain ∑ 𝑓𝑑2
.
Thus the only difference in procedurein case of continuous series is to find mid-
points of the various classes.
Example--Calculate Standard deviation from the following data:
Age 20-25 25-30 30-35 35-40 40-45 45-50
No. of Persons 170 110 80 45 40 35
Solution:
Take assumed average = 32.5
Calculation of Standard deviation
Age Mid
point(m)
No. of
Persons (f)
𝒅 =
𝒎 − 𝟑𝟐. 𝟓
𝟓
𝒇𝒅 𝒇𝒅 𝟐
20-25 22.5 170 -2 -340 680
25-30 27.5 110 -1 -110 110
30-35 32.5 80 0 0 0
35-40 37.5 45 +1 +45 45
Unit-3 FREQUAENCY DISTRIBUTION
RAI UNIVERSITY, AHMEDABAD 49
40-45 42.5 40 +2 +80 160
45-50 47.5 35 +3 +105 315
𝑵 = 𝟒𝟖𝟎 ∑ 𝒇𝒅 =-220 ∑ 𝒇𝒅 𝟐
=1310
 𝜎 = √
∑ 𝑓𝑑2
𝑁
− (
∑ 𝑓𝑑
𝑁
)
2
× 𝑖
 𝜎 = √1310
480
− (
−220
480
)
2
× 5
 𝜎 = √2.279− 0.21 × 5
 𝜎 = √2.519× 5
 𝜎 = 1.587× 5 = 7.936
14.1Coefficientof Variation:
The standard deviation discussed aboveis an absolute measure of dispersion. The
corresponding relative measure is known as the Coefficient of variation.
This measure developed by karl pearson is the most commonly used measure of
relative variation. It is used in such problems where we cant to compare the
variability of two or more than two series.
That series for which the coefficient of variation is greater is said to be more
variable or conversely less consistent,less uniform,less srable .
On the other hand , the series for which coefficient of variation is less is said to be
less variable or more consistent,more uniform,more stable .
Coefficient of variation is denoted by C.V. and is obtained as follows:
𝐶𝑜𝑒𝑓𝑓𝑖𝑐𝑖𝑒𝑛𝑡 𝑜𝑓 𝑣𝑎𝑟𝑖𝑎𝑡𝑖𝑜𝑛 𝑜𝑟 𝐶. 𝑉. =
𝜎
𝐴
× 100
Where, 𝜎 = standard deviation, 𝐴 = 𝑎𝑟𝑖𝑡ℎ𝑚𝑒𝑡𝑖𝑐 𝑀𝑒𝑎𝑛
Unit-3 FREQUAENCY DISTRIBUTION
RAI UNIVERSITY, AHMEDABAD 50
Example--From the prices of shares of 𝑿 and 𝒀 below find out which is more
stable in value:
𝑿 35 54 52 53 56 58 52 50 51 49
𝒀 108 107 105 105 106 107 104 103 104 101
Solution:
In order to find out which shares are more stable, we have to compare coefficient
of variations.
Calculation of Coefficient of Variation
𝑿 𝒙 = (𝑿 − 𝑨) 𝒙 𝟐 𝒀 𝒚 = (𝒀 − 𝑨) 𝒚 𝟐
35 -16 256 108 +3 9
54 +3 9 107 +2 4
52 +1 1 105 0 0
53 +2 4 105 0 0
56 +5 25 106 +1 1
58 +7 49 107 +2 4
52 +1 1 104 -1 1
50 -1 1 103 -2 4
51 0 0 104 -1 1
49 -2 4 101 -4 16
∑ 𝑿 =510 ∑ 𝒙 =0 ∑ 𝒙 𝟐
=350 ∑ 𝒀 =1050 ∑ 𝒚=0 ∑ 𝒚 𝟐
=40
Coefficientof variation 𝑿:
 𝐶. 𝑉. =
𝜎
𝐴
× 100
 Here 𝐴 =
∑ 𝑋
𝑁
=
510
10
= 51
Unit-3 FREQUAENCY DISTRIBUTION
RAI UNIVERSITY, AHMEDABAD 51
 𝜎 = √
∑ 𝑥2
𝑁
= √
350
10
= 5.916
 ∴ 𝐶. 𝑉. =
𝜎
𝐴
× 100 =
5.196
51
× 100
 ∴ 𝐶. 𝑉. = 11.6
Coefficientof variation 𝒀:
 𝐶. 𝑉. =
𝜎
𝐴
× 100
 Here 𝐴 =
∑ 𝑦
𝑁
=
1050
10
= 105
 𝜎 = √
∑ 𝑦2
𝑁
= √
40
10
= 2
 ∴ 𝐶. 𝑉. =
𝜎
𝐴
× 100 =
2
105
× 100
 ∴ 𝐶. 𝑉. = 1.905
Since Coefficient of variation is much less in case of shares 𝑌, Hence they are
more stable in value.
EXERCISE
Q-1 Evaluate the following Questions:
(1). Find the Mean, Median and Mode of the following data.
Cost 10-20 20-30 30-40 40-50 50-60
Items in a
group
4 5 3 6 3
(2).Find the Mean, Median and Mode of the following distribution:
Class
Interval
0-10 10-20 20-30 30-40 40-50 50-60 60-70 70-80
Frequency 5 9 8 12 28 20 12 11
(3).On his first 5 biology tests, Bob received the following scores: 72, 86, 92, 63,
and 77. What test scoremust Bob earn on his sixth test so that his average (mean
score)for all six tests will be 80? Show how you arrived at your answer.
Q-2 Evaluate the following Questions:
Unit-3 FREQUAENCY DISTRIBUTION
RAI UNIVERSITY, AHMEDABAD 52
(1). Calculate quartile deviation and it coefficient from the following data:
Wages in Rupees per
Day
Less then
35
35-
37
38-40 41-43 Over 43
Number of Wage
earners
14 62 99 18 7
(2). Calculate Mean Deviation for the following data
Size 0-10 10-20 20-30 30-40 40-50 50-60 60-70
Frequency 7 12 18 25 16 14 8
(3). The owner of a restaurant is interested in how much people spend at the
restaurant. He examines 10 randomly selected receipts for parties of four and write
down the following data: 44, 50, 38, 96, 42, 47, 40,39, 46, 50.
Find mean, standard deviation and variance.
15. Referenc Book andWebsite Name:
1. http://www.emathzone.com/tutorials/basic-statistics/collection-of-statistical-
data.html
2. http://wizznotes.com/mathematics/statistics/class-limits-boundaries-and-
intervals
3. http://tistats.com/definitions/class-width/
4. http://mathespk.blogspot.in/2011/10/frequency-density.html
5. http://www.mathsisfun.com/definitions/relative-frequency.html
6. http://www.mathsisfun.com/definitions/cumulative-frequency.html
7. http://www.mathgoodies.com/lessons/vol8/mode.html
8. Statistical Methods by S.P.Gupta.
Unit-3 FREQUAENCY DISTRIBUTION
RAI UNIVERSITY, AHMEDABAD 53

Weitere ähnliche Inhalte

Was ist angesagt?

Computer Aided Design: Layout Compaction
Computer Aided Design: Layout CompactionComputer Aided Design: Layout Compaction
Computer Aided Design: Layout Compaction
Team-VLSI-ITMU
 

Was ist angesagt? (20)

MHC, STRUCTURE AND FUNCTION
MHC, STRUCTURE AND FUNCTIONMHC, STRUCTURE AND FUNCTION
MHC, STRUCTURE AND FUNCTION
 
B Lymphocytes
B Lymphocytes B Lymphocytes
B Lymphocytes
 
Digital Design Session 24
Digital Design Session 24Digital Design Session 24
Digital Design Session 24
 
Synchronization
SynchronizationSynchronization
Synchronization
 
dot plot analysis
dot plot analysisdot plot analysis
dot plot analysis
 
UNIT-II : SEQUENTIAL CIRCUIT DESIGN
UNIT-II  : SEQUENTIAL CIRCUIT DESIGN UNIT-II  : SEQUENTIAL CIRCUIT DESIGN
UNIT-II : SEQUENTIAL CIRCUIT DESIGN
 
Logic gates and Boolean Algebra_VSG
Logic gates and Boolean Algebra_VSGLogic gates and Boolean Algebra_VSG
Logic gates and Boolean Algebra_VSG
 
Power control in 3 g
Power control in 3 gPower control in 3 g
Power control in 3 g
 
Generation of fm
Generation of fmGeneration of fm
Generation of fm
 
Digital Data, Digital Signal | Scrambling Techniques
Digital Data, Digital Signal | Scrambling TechniquesDigital Data, Digital Signal | Scrambling Techniques
Digital Data, Digital Signal | Scrambling Techniques
 
Computer Aided Design: Layout Compaction
Computer Aided Design: Layout CompactionComputer Aided Design: Layout Compaction
Computer Aided Design: Layout Compaction
 
Combinational circuit
Combinational circuitCombinational circuit
Combinational circuit
 
EMBL- European Molecular Biology Laboratory
EMBL- European Molecular Biology LaboratoryEMBL- European Molecular Biology Laboratory
EMBL- European Molecular Biology Laboratory
 
Lymphoid organs ppt
Lymphoid organs  pptLymphoid organs  ppt
Lymphoid organs ppt
 
Modulation Techniques for Mobile Radio
Modulation Techniques for Mobile RadioModulation Techniques for Mobile Radio
Modulation Techniques for Mobile Radio
 
Phylogenetic analysis
Phylogenetic analysisPhylogenetic analysis
Phylogenetic analysis
 
Application of bio informatics by sohail
Application of bio informatics by sohailApplication of bio informatics by sohail
Application of bio informatics by sohail
 
Error control coding techniques
Error control coding techniquesError control coding techniques
Error control coding techniques
 
BCR
BCR BCR
BCR
 
Sequence alignment
Sequence alignmentSequence alignment
Sequence alignment
 

Andere mochten auch

MITCOE 2011-12 conm-submission
MITCOE 2011-12 conm-submissionMITCOE 2011-12 conm-submission
MITCOE 2011-12 conm-submission
Ashutosh Katti
 

Andere mochten auch (20)

MCA_UNIT-4_Computer Oriented Numerical Statistical Methods
MCA_UNIT-4_Computer Oriented Numerical Statistical MethodsMCA_UNIT-4_Computer Oriented Numerical Statistical Methods
MCA_UNIT-4_Computer Oriented Numerical Statistical Methods
 
MCA_UNIT-2_Computer Oriented Numerical Statistical Methods
MCA_UNIT-2_Computer Oriented Numerical Statistical MethodsMCA_UNIT-2_Computer Oriented Numerical Statistical Methods
MCA_UNIT-2_Computer Oriented Numerical Statistical Methods
 
Course pack unit 5
Course pack unit 5Course pack unit 5
Course pack unit 5
 
MCA_UNIT-1_Computer Oriented Numerical Statistical Methods
MCA_UNIT-1_Computer Oriented Numerical Statistical MethodsMCA_UNIT-1_Computer Oriented Numerical Statistical Methods
MCA_UNIT-1_Computer Oriented Numerical Statistical Methods
 
Mm unit 4point2
Mm unit 4point2Mm unit 4point2
Mm unit 4point2
 
Mca 2 sem u-1 iintroduction
Mca 2 sem u-1 iintroductionMca 2 sem u-1 iintroduction
Mca 2 sem u-1 iintroduction
 
Module 2 statistics
Module 2   statisticsModule 2   statistics
Module 2 statistics
 
MITCOE 2011-12 conm-submission
MITCOE 2011-12 conm-submissionMITCOE 2011-12 conm-submission
MITCOE 2011-12 conm-submission
 
A time study in numerical methods programming
A time study in numerical methods programmingA time study in numerical methods programming
A time study in numerical methods programming
 
Bisection & Regual falsi methods
Bisection & Regual falsi methodsBisection & Regual falsi methods
Bisection & Regual falsi methods
 
B.Tech-II_Unit-IV
B.Tech-II_Unit-IVB.Tech-II_Unit-IV
B.Tech-II_Unit-IV
 
B.tech ii unit-4 material vector differentiation
B.tech ii unit-4 material vector differentiationB.tech ii unit-4 material vector differentiation
B.tech ii unit-4 material vector differentiation
 
BSC_Computer Science_Discrete Mathematics_Unit-I
BSC_Computer Science_Discrete Mathematics_Unit-IBSC_Computer Science_Discrete Mathematics_Unit-I
BSC_Computer Science_Discrete Mathematics_Unit-I
 
B.Tech-II_Unit-V
B.Tech-II_Unit-VB.Tech-II_Unit-V
B.Tech-II_Unit-V
 
Btech_II_ engineering mathematics_unit4
Btech_II_ engineering mathematics_unit4Btech_II_ engineering mathematics_unit4
Btech_II_ engineering mathematics_unit4
 
B.tech ii unit-5 material vector integration
B.tech ii unit-5 material vector integrationB.tech ii unit-5 material vector integration
B.tech ii unit-5 material vector integration
 
merged_document
merged_documentmerged_document
merged_document
 
BCA_Semester-II-Discrete Mathematics_unit-iv Graph theory
BCA_Semester-II-Discrete Mathematics_unit-iv Graph theoryBCA_Semester-II-Discrete Mathematics_unit-iv Graph theory
BCA_Semester-II-Discrete Mathematics_unit-iv Graph theory
 
B.Tech-II_Unit-I
B.Tech-II_Unit-IB.Tech-II_Unit-I
B.Tech-II_Unit-I
 
Unit 1 Introduction
Unit 1 IntroductionUnit 1 Introduction
Unit 1 Introduction
 

Ähnlich wie MCA_UNIT-3_Computer Oriented Numerical Statistical Methods

Data organization and presentation (statistics for research)
Data organization and presentation (statistics for research)Data organization and presentation (statistics for research)
Data organization and presentation (statistics for research)
Harve Abella
 

Ähnlich wie MCA_UNIT-3_Computer Oriented Numerical Statistical Methods (20)

Classification of data
Classification of dataClassification of data
Classification of data
 
Chapter 4 MMW.pdf
Chapter 4 MMW.pdfChapter 4 MMW.pdf
Chapter 4 MMW.pdf
 
Ch 3 DATA.doc
Ch 3 DATA.docCh 3 DATA.doc
Ch 3 DATA.doc
 
Statistical Techniques in Business and Economics 15th Edition Lind Test Bank
Statistical Techniques in Business and Economics 15th Edition Lind Test BankStatistical Techniques in Business and Economics 15th Edition Lind Test Bank
Statistical Techniques in Business and Economics 15th Edition Lind Test Bank
 
Unit 1 - Statistics (Part 1).pptx
Unit 1 - Statistics (Part 1).pptxUnit 1 - Statistics (Part 1).pptx
Unit 1 - Statistics (Part 1).pptx
 
Nota Bab 1 JF608
Nota Bab 1 JF608Nota Bab 1 JF608
Nota Bab 1 JF608
 
Principlles of statistics [amar mamusta amir]
Principlles of statistics [amar mamusta amir]Principlles of statistics [amar mamusta amir]
Principlles of statistics [amar mamusta amir]
 
Data organization and presentation (statistics for research)
Data organization and presentation (statistics for research)Data organization and presentation (statistics for research)
Data organization and presentation (statistics for research)
 
1.0 Descriptive statistics.pdf
1.0 Descriptive statistics.pdf1.0 Descriptive statistics.pdf
1.0 Descriptive statistics.pdf
 
Lecture notes on STS 102
Lecture notes on STS 102Lecture notes on STS 102
Lecture notes on STS 102
 
CH1.pdf
CH1.pdfCH1.pdf
CH1.pdf
 
Ch1
Ch1Ch1
Ch1
 
Edited economic statistics note
Edited economic statistics noteEdited economic statistics note
Edited economic statistics note
 
lecture 1 Slides.pptx
lecture 1 Slides.pptxlecture 1 Slides.pptx
lecture 1 Slides.pptx
 
Principlles of statistics
Principlles of statisticsPrinciplles of statistics
Principlles of statistics
 
Lesson 5 data presentation
Lesson 5 data presentationLesson 5 data presentation
Lesson 5 data presentation
 
Lesson 002
Lesson 002Lesson 002
Lesson 002
 
Classification and tabulation of data
Classification and tabulation of dataClassification and tabulation of data
Classification and tabulation of data
 
001
001001
001
 
Bcs 040 Descriptive Statistics
Bcs 040 Descriptive StatisticsBcs 040 Descriptive Statistics
Bcs 040 Descriptive Statistics
 

Mehr von Rai University

Bsc agri 2 pae u-4.4 publicrevenue-presentation-130208082149-phpapp02
Bsc agri  2 pae  u-4.4 publicrevenue-presentation-130208082149-phpapp02Bsc agri  2 pae  u-4.4 publicrevenue-presentation-130208082149-phpapp02
Bsc agri 2 pae u-4.4 publicrevenue-presentation-130208082149-phpapp02
Rai University
 

Mehr von Rai University (20)

Brochure Rai University
Brochure Rai University Brochure Rai University
Brochure Rai University
 
Mm unit 4point1
Mm unit 4point1Mm unit 4point1
Mm unit 4point1
 
Mm unit 4point3
Mm unit 4point3Mm unit 4point3
Mm unit 4point3
 
Mm unit 3point2
Mm unit 3point2Mm unit 3point2
Mm unit 3point2
 
Mm unit 3point1
Mm unit 3point1Mm unit 3point1
Mm unit 3point1
 
Mm unit 2point2
Mm unit 2point2Mm unit 2point2
Mm unit 2point2
 
Mm unit 2 point 1
Mm unit 2 point 1Mm unit 2 point 1
Mm unit 2 point 1
 
Mm unit 1point3
Mm unit 1point3Mm unit 1point3
Mm unit 1point3
 
Mm unit 1point2
Mm unit 1point2Mm unit 1point2
Mm unit 1point2
 
Mm unit 1point1
Mm unit 1point1Mm unit 1point1
Mm unit 1point1
 
Bdft ii, tmt, unit-iii, dyeing & types of dyeing,
Bdft ii, tmt, unit-iii,  dyeing & types of dyeing,Bdft ii, tmt, unit-iii,  dyeing & types of dyeing,
Bdft ii, tmt, unit-iii, dyeing & types of dyeing,
 
Bsc agri 2 pae u-4.4 publicrevenue-presentation-130208082149-phpapp02
Bsc agri  2 pae  u-4.4 publicrevenue-presentation-130208082149-phpapp02Bsc agri  2 pae  u-4.4 publicrevenue-presentation-130208082149-phpapp02
Bsc agri 2 pae u-4.4 publicrevenue-presentation-130208082149-phpapp02
 
Bsc agri 2 pae u-4.3 public expenditure
Bsc agri  2 pae  u-4.3 public expenditureBsc agri  2 pae  u-4.3 public expenditure
Bsc agri 2 pae u-4.3 public expenditure
 
Bsc agri 2 pae u-4.2 public finance
Bsc agri  2 pae  u-4.2 public financeBsc agri  2 pae  u-4.2 public finance
Bsc agri 2 pae u-4.2 public finance
 
Bsc agri 2 pae u-4.1 introduction
Bsc agri  2 pae  u-4.1 introductionBsc agri  2 pae  u-4.1 introduction
Bsc agri 2 pae u-4.1 introduction
 
Bsc agri 2 pae u-3.3 inflation
Bsc agri  2 pae  u-3.3  inflationBsc agri  2 pae  u-3.3  inflation
Bsc agri 2 pae u-3.3 inflation
 
Bsc agri 2 pae u-3.2 introduction to macro economics
Bsc agri  2 pae  u-3.2 introduction to macro economicsBsc agri  2 pae  u-3.2 introduction to macro economics
Bsc agri 2 pae u-3.2 introduction to macro economics
 
Bsc agri 2 pae u-3.1 marketstructure
Bsc agri  2 pae  u-3.1 marketstructureBsc agri  2 pae  u-3.1 marketstructure
Bsc agri 2 pae u-3.1 marketstructure
 
Bsc agri 2 pae u-3 perfect-competition
Bsc agri  2 pae  u-3 perfect-competitionBsc agri  2 pae  u-3 perfect-competition
Bsc agri 2 pae u-3 perfect-competition
 
Bsc agri 2 pae u-2.4 different forms of business organizing
Bsc agri  2 pae  u-2.4  different forms of business organizingBsc agri  2 pae  u-2.4  different forms of business organizing
Bsc agri 2 pae u-2.4 different forms of business organizing
 

Kürzlich hochgeladen

Seal of Good Local Governance (SGLG) 2024Final.pptx
Seal of Good Local Governance (SGLG) 2024Final.pptxSeal of Good Local Governance (SGLG) 2024Final.pptx
Seal of Good Local Governance (SGLG) 2024Final.pptx
negromaestrong
 
The basics of sentences session 3pptx.pptx
The basics of sentences session 3pptx.pptxThe basics of sentences session 3pptx.pptx
The basics of sentences session 3pptx.pptx
heathfieldcps1
 
1029-Danh muc Sach Giao Khoa khoi 6.pdf
1029-Danh muc Sach Giao Khoa khoi  6.pdf1029-Danh muc Sach Giao Khoa khoi  6.pdf
1029-Danh muc Sach Giao Khoa khoi 6.pdf
QucHHunhnh
 
Jual Obat Aborsi Hongkong ( Asli No.1 ) 085657271886 Obat Penggugur Kandungan...
Jual Obat Aborsi Hongkong ( Asli No.1 ) 085657271886 Obat Penggugur Kandungan...Jual Obat Aborsi Hongkong ( Asli No.1 ) 085657271886 Obat Penggugur Kandungan...
Jual Obat Aborsi Hongkong ( Asli No.1 ) 085657271886 Obat Penggugur Kandungan...
ZurliaSoop
 

Kürzlich hochgeladen (20)

Third Battle of Panipat detailed notes.pptx
Third Battle of Panipat detailed notes.pptxThird Battle of Panipat detailed notes.pptx
Third Battle of Panipat detailed notes.pptx
 
Spatium Project Simulation student brief
Spatium Project Simulation student briefSpatium Project Simulation student brief
Spatium Project Simulation student brief
 
SOC 101 Demonstration of Learning Presentation
SOC 101 Demonstration of Learning PresentationSOC 101 Demonstration of Learning Presentation
SOC 101 Demonstration of Learning Presentation
 
Magic bus Group work1and 2 (Team 3).pptx
Magic bus Group work1and 2 (Team 3).pptxMagic bus Group work1and 2 (Team 3).pptx
Magic bus Group work1and 2 (Team 3).pptx
 
ICT role in 21st century education and it's challenges.
ICT role in 21st century education and it's challenges.ICT role in 21st century education and it's challenges.
ICT role in 21st century education and it's challenges.
 
Seal of Good Local Governance (SGLG) 2024Final.pptx
Seal of Good Local Governance (SGLG) 2024Final.pptxSeal of Good Local Governance (SGLG) 2024Final.pptx
Seal of Good Local Governance (SGLG) 2024Final.pptx
 
Application orientated numerical on hev.ppt
Application orientated numerical on hev.pptApplication orientated numerical on hev.ppt
Application orientated numerical on hev.ppt
 
Accessible Digital Futures project (20/03/2024)
Accessible Digital Futures project (20/03/2024)Accessible Digital Futures project (20/03/2024)
Accessible Digital Futures project (20/03/2024)
 
This PowerPoint helps students to consider the concept of infinity.
This PowerPoint helps students to consider the concept of infinity.This PowerPoint helps students to consider the concept of infinity.
This PowerPoint helps students to consider the concept of infinity.
 
Mixin Classes in Odoo 17 How to Extend Models Using Mixin Classes
Mixin Classes in Odoo 17  How to Extend Models Using Mixin ClassesMixin Classes in Odoo 17  How to Extend Models Using Mixin Classes
Mixin Classes in Odoo 17 How to Extend Models Using Mixin Classes
 
Food safety_Challenges food safety laboratories_.pdf
Food safety_Challenges food safety laboratories_.pdfFood safety_Challenges food safety laboratories_.pdf
Food safety_Challenges food safety laboratories_.pdf
 
Basic Civil Engineering first year Notes- Chapter 4 Building.pptx
Basic Civil Engineering first year Notes- Chapter 4 Building.pptxBasic Civil Engineering first year Notes- Chapter 4 Building.pptx
Basic Civil Engineering first year Notes- Chapter 4 Building.pptx
 
The basics of sentences session 3pptx.pptx
The basics of sentences session 3pptx.pptxThe basics of sentences session 3pptx.pptx
The basics of sentences session 3pptx.pptx
 
Micro-Scholarship, What it is, How can it help me.pdf
Micro-Scholarship, What it is, How can it help me.pdfMicro-Scholarship, What it is, How can it help me.pdf
Micro-Scholarship, What it is, How can it help me.pdf
 
ComPTIA Overview | Comptia Security+ Book SY0-701
ComPTIA Overview | Comptia Security+ Book SY0-701ComPTIA Overview | Comptia Security+ Book SY0-701
ComPTIA Overview | Comptia Security+ Book SY0-701
 
TỔNG ÔN TẬP THI VÀO LỚP 10 MÔN TIẾNG ANH NĂM HỌC 2023 - 2024 CÓ ĐÁP ÁN (NGỮ Â...
TỔNG ÔN TẬP THI VÀO LỚP 10 MÔN TIẾNG ANH NĂM HỌC 2023 - 2024 CÓ ĐÁP ÁN (NGỮ Â...TỔNG ÔN TẬP THI VÀO LỚP 10 MÔN TIẾNG ANH NĂM HỌC 2023 - 2024 CÓ ĐÁP ÁN (NGỮ Â...
TỔNG ÔN TẬP THI VÀO LỚP 10 MÔN TIẾNG ANH NĂM HỌC 2023 - 2024 CÓ ĐÁP ÁN (NGỮ Â...
 
1029-Danh muc Sach Giao Khoa khoi 6.pdf
1029-Danh muc Sach Giao Khoa khoi  6.pdf1029-Danh muc Sach Giao Khoa khoi  6.pdf
1029-Danh muc Sach Giao Khoa khoi 6.pdf
 
Jual Obat Aborsi Hongkong ( Asli No.1 ) 085657271886 Obat Penggugur Kandungan...
Jual Obat Aborsi Hongkong ( Asli No.1 ) 085657271886 Obat Penggugur Kandungan...Jual Obat Aborsi Hongkong ( Asli No.1 ) 085657271886 Obat Penggugur Kandungan...
Jual Obat Aborsi Hongkong ( Asli No.1 ) 085657271886 Obat Penggugur Kandungan...
 
psychiatric nursing HISTORY COLLECTION .docx
psychiatric  nursing HISTORY  COLLECTION  .docxpsychiatric  nursing HISTORY  COLLECTION  .docx
psychiatric nursing HISTORY COLLECTION .docx
 
How to Give a Domain for a Field in Odoo 17
How to Give a Domain for a Field in Odoo 17How to Give a Domain for a Field in Odoo 17
How to Give a Domain for a Field in Odoo 17
 

MCA_UNIT-3_Computer Oriented Numerical Statistical Methods

  • 1. Unit-3 FREQUAENCY DISTRIBUTION RAI UNIVERSITY, AHMEDABAD 1 Course: MCA Subject: Computer Oriented Numerical Statistical Methods Unit-3 RAI UNIVERSITY, AHMEDABAD
  • 2. Unit-3 FREQUAENCY DISTRIBUTION RAI UNIVERSITY, AHMEDABAD 2 Unit-III- Frequaency Distribution Sr. No. Name of the Topic Page No. 1 Introduction, Collection of data, Classification of data 2 2 Introduction to frequency distribution , Class Limit, , Class Interval,Class frequency, Class mark, Class Boundaries, Width of a class 5 3 Frequency density, Relative frequency, Percentage frequency, Cumulative frequency 9 4 Introduction, Arithmetic Mean, Mean forDiscrete frequency distribution, Mean for Continuous frequency distribution,Weighted Arithmetic Mean 11 5 Properties of A.M., Merits & De merits of A.M., 22 6 Median for raw data, Discrete frequency distribution, Continuous frequency distribution, 23 7 Merits and demerits of Median 26 8 Mode for raw data, D.f.s., c.f.s., 27 9 Merits & demerits of mode 30 10 Introduction, Range, coefficient of range,Merit & Demerit of Range 30 11 Quartiles, Quartiles deviations, coefficient of quartile deviations 31 12 Mean deviation and coefficient of mean deviation,Merit and Demerit of Mean Deviation 33 13 Standard Deviation and variance for all types of frequency distribution 38 14 Coefficient of variation 48
  • 3. Unit-3 FREQUAENCY DISTRIBUTION RAI UNIVERSITY, AHMEDABAD 3 1.1 Introduction: A sequence of observation, made on a set of objects included in the sample drawn from population is known as statistical data. (1). Ungrouped data Data which have been arranged in a systematic order are called raw data or ungrouped data. (2) .Grouped data Data presented in the form of frequency distribution is called grouped data. 1.2Collectionof Data: The first step in any enquiry (investigation) is collection of data. The data may be collected for the whole population or for a sample only. It is mostly collected on sample basis. Collection of data is very difficult job. The enumerator or investigator is the well trained personwho collects the statistical data. The respondents (information) are the persons whom the information is collected. 1.3 ClassificationofData: Data classification is the process oforganizing data into categories for its most effective and efficient use. A well-planned data classification system makes essential data easy to find and retrieve. Written procedures and guidelines for data classification should define what categories and criteria the organization will use to classify data and specify the roles and responsibilities of employees within the organization regarding data stewardship. Once a data-classification scheme has been created, security standards that specify appropriate handling practices for each category and storage standards that define the data's lifecycle requirements should be addressed. Here is an example of what a data classification scheme might look like:
  • 4. Unit-3 FREQUAENCY DISTRIBUTION RAI UNIVERSITY, AHMEDABAD 4 Category4: Highly sensitive corporateand customer data that if disclosed could put the organization at financial or legal risk. Example: Employee social security numbers, customer credit card numbers Category3: Sensitive internal data that if disclosed could negatively affect operations. Example: Contracts with third-party suppliers, employee reviews Category2: Internal data that is not meant for public disclosure. Example: Sales contest rules, organizational charts Category1: Data that may be freely disclosed with the public. Example: Contact information, price lists 1.3.1 Types of Data: There are two types (sources) for the collection of data. (1) Primary Data (2) SecondaryData (1) Primary Data: The primary data are the first hand information collected, compiled and published by organization for some purpose. They are most original data in character and have not undergone any sortof statistical treatment. Example: Population census reports are primary data because these are collected, complied and published by the population census organization. (2) SecondaryData: The secondarydata are the second hand information which are already collected by some one (organization) for some purposeand are available for the present study. The secondarydata are not pure in character and have undergone some treatment at least once. Example: Economics survey of England is secondarydata because these are collected by more than one organization like Bureau of statistics, Board of Revenue, the Banks etc…
  • 5. Unit-3 FREQUAENCY DISTRIBUTION RAI UNIVERSITY, AHMEDABAD 5 1.3.2DifferencebetweenPrimary and SecondaryData: The difference between primary and secondary data is only a change of hand. The primary data are the first hand data information which is directly collected form one source. They are most original data in character and have not undergone any sort of statistical treatment while the secondarydata are obtained from some other sources or agencies. They are not pure in character and have undergone some treatment at least once. For Example: Supposewe interested to find the average age of MS students. We collect the age’s data by two methods; either by directly collecting from each student himself personally or getting their ages from the university record. The data collected by the direct personal investigation is called primary data and the data obtained from the university record is called secondary data. 1.3.3 Types of Classification: 1. Geographical Classification: i.e. Area wise, e.g. cities, districts, etc. 2. Chronological Classification: i.e. on the basis of time 3. Qualitative Classification: i.e. according to some attributes 4. Quantitative Classification: i.e. in terms of magnitudes Quantitative classification refers to the classification of data according to some characteristics that can be measured, such as height, weight, income, sales, profits etc. In this type of classification , there are two elements namely (i). the variable (ii). The frequency: Frequency is how often something occurs.
  • 6. Unit-3 FREQUAENCY DISTRIBUTION RAI UNIVERSITY, AHMEDABAD 6 2.1Definition:frequency Distribution By counting frequencies we can make a Frequency Distribution table. There are two types of frequency distribution: (a). Discrete frequency distribution. (b). Continuous frequency Distribution. Example: Newspapers These are the numbers of newspapers sold at a local shop over the last 10 days: 22, 20, 18, 23, 20, 25, 22, 20, 18, 20 Let us count how many of each number there is: Here Table-A is a example of discrete frequency distribution Table-B is a example of continuous frequency distribution. TABLE-A Papers Sold Frequency 18 2 19 0 20 4 21 0 22 2 23 1 24 0 25 1 It is also possible to group the values. Here they are in grouped : Table-B
  • 7. Unit-3 FREQUAENCY DISTRIBUTION RAI UNIVERSITY, AHMEDABAD 7 Papers Sold Frequency 15-19 2 20-24 7 25-29 1 2.2ClassLimit: Class limits are the smallest and largest observations (data, events etc) in each class. Therefore, each class has two limits: a lower and upper. Example for, In the above Table-B For the first class 15-19 The Lower class limit is =15 The Upper class limit is = 19 2.3Class Interval: The difference between the upper and lower limit of a class is known as class interval of that class. Table-B Papers Sold Frequency 15-19 2 20-24 7 25-29 1 For example: ForTable-B in the class 15-19 the class interval = 4 (i.e. 19-15) A simple formula to obtain the estimate of appropriate class interval, i.e. 𝑖 = 𝐿−𝑆 𝑘 where 𝐿 = largest item, S= Smallest item, K = no. of classes
  • 8. Unit-3 FREQUAENCY DISTRIBUTION RAI UNIVERSITY, AHMEDABAD 8 2.3.1 Example:If the salary of 100 employees in a commercialundertaking varied between Rs.10,000and Rs. 30,000and we want to form 10 classes,then Find the class interval. Solution: 𝑖 = 𝐿−𝑆 𝑘  Here 𝐿 = 30,000, 𝑆 = 10,000, 𝑘 = 10  𝑖 = 30,000−10,000 10 = 20,000 10 = 2000 2.4 Class Frequency: The number of observations correspondingto a particular class is known as the frequency of that class or the class frequency. Table-B Papers Sold Frequency 15-19 2 20-24 7 25-29 1 For example, In the Table-B the class 20-24 has frequency 7. i.e there are 7 days in which no. of papers sold is between 20-24. If we add together frequencies of all individual classes, we obtain the total frequency. The total frequency of Table-B = 2+7+1=10 2.5 Class mark or class mid point: It is the value lying half-way between the lower and upper class limits of a class interval.
  • 9. Unit-3 FREQUAENCY DISTRIBUTION RAI UNIVERSITY, AHMEDABAD 9 ∴ 𝑐𝑙𝑎𝑠𝑠 𝑚𝑎𝑟𝑘 = 𝑈𝑝𝑝𝑒𝑟 𝑙𝑖𝑚𝑖𝑡 𝑜𝑓 𝑡ℎ𝑒 𝑐𝑙𝑎𝑠𝑠+𝑙𝑜𝑤𝑒𝑟 𝑙𝑖𝑚𝑖𝑡 𝑜𝑓 𝑡ℎ𝑒 𝑐𝑙𝑎𝑠𝑠 2 2.6 Class Boundaries: Class Boundaries are the midpoints between the upper class limit of a class and the lower class limit of the next class in the sequence. Therefore, each class has an upper and lower class boundary. For Example: Table-B Papers Sold Frequency 15-19 2 20-24 7 25-29 1 For the first class in table-B , 15 – 19 The lower class boundary is the midpoint between 14 and 15, that is 14.5 The upper class boundary is the midpoint between 19and 20, that is 19.5 2.7 Width of a Class: Difference between two consecutive lower class limits Difference between two consecutive upper class limits Example for, Table-B Papers Sold Frequency 15-19 2 20-24 7 25-29 1 Difference between two consecutive lower class limits 20-15 = 5
  • 10. Unit-3 FREQUAENCY DISTRIBUTION RAI UNIVERSITY, AHMEDABAD 10 Difference between two consecutive upper class limits 24-19 = 5 ∴Class width=5 3.1Frequency Density: 𝐹𝑟𝑒𝑞𝑢𝑒𝑛𝑐𝑦 𝑑𝑒𝑛𝑠𝑖𝑡𝑦 = 𝑓𝑟𝑒𝑞𝑢𝑒𝑛𝑐𝑦 𝑐𝑙𝑎𝑠𝑠 𝑤𝑖𝑑𝑡ℎ Frequency density is use to draw histogram . The following table shows the ages of 25 children on a school bus: Age Frequency 5-10 6 11-15 15 16-17 4 > 17 0 To draw the histogram we need frequency density: For class 5-10 𝐹𝑟𝑒𝑞𝑢𝑒𝑛𝑐𝑦 𝑑𝑒𝑛𝑠𝑖𝑡𝑦 = 𝑓𝑟𝑒𝑞𝑢𝑒𝑛𝑐𝑦 𝑐𝑙𝑎𝑠𝑠 𝑤𝑖𝑑𝑡ℎ = 6 6 = 1 For class 11-15 𝐹𝑟𝑒𝑞𝑢𝑒𝑛𝑐𝑦 𝑑𝑒𝑛𝑠𝑖𝑡𝑦 = 𝑓𝑟𝑒𝑞𝑢𝑒𝑛𝑐𝑦 𝑐𝑙𝑎𝑠𝑠 𝑤𝑖𝑑𝑡ℎ = 15 5 = 3
  • 11. Unit-3 FREQUAENCY DISTRIBUTION RAI UNIVERSITY, AHMEDABAD 11 3.2 Relative frequency: Relative frequency is the ratio of the number of times an event occurs to the number of occasions on which it might occurin the same period. In other words, how often something happens divided by all outcomes. Example: if your team has won 9 games from a total of 12 games played: * the Frequency of winning is 9 * the Relative Frequency of winning is 9 12 = 75% 3.3 Percentagefrequency: Percentage frequency that means calculate percentage of given frequency. Percentage frequency= 100×𝑓𝑟𝑒𝑞𝑢𝑒𝑛𝑐𝑦 𝑜𝑓 𝑔𝑖𝑣𝑒𝑛 𝑐𝑙𝑎𝑠𝑠 𝑡𝑜𝑡𝑎𝑙 𝑓𝑟𝑒𝑞𝑢𝑒𝑛𝑐𝑦 Papers Sold Frequency 15-19 2 20-24 7 25-29 1 Forclass 15-19 frequency is = 2 ∴ Percentage frequency for class 15-19 = 100×2 10 = 20%
  • 12. Unit-3 FREQUAENCY DISTRIBUTION RAI UNIVERSITY, AHMEDABAD 12 3.4Cumulative frequency: The total of a frequency and all frequencies so far in a frequency distribution. It is the 'running total' of frequencies. 4.1Introduction: A measure of central tendency is a single value that attempts to describe a set of data by identifying the central position within that set of data. Measures of central tendency are sometimes called measures of central location. They are also classed as summary statistics. The mean (often called the average) is most likely the measure of central tendency that you are most familiar with, but there are others, such as the median and the mode. The mean, median and mode are all valid measures of central tendency. 4.2Arithmetic Mean: The most popular and widely used measure of representing the entire data by one value is what most laymen call an average and what the statisticians call the
  • 13. Unit-3 FREQUAENCY DISTRIBUTION RAI UNIVERSITY, AHMEDABAD 13 arithmetic mean. Its value is obtained by adding together all the items and by dividing this total by the number of items. Arithmetic mean may either be i. Simple arithmetic mean, or ii. Weighted arithmetic mean First of all we have to discuss about Simple Arithmetic Mean. There are two method for finding simple arithmetic Mean : 1. Direct Method 2. Short-cut Method 4.2.1 DirectMethod for finding Arithmetic Mean : 𝑿̅ = 𝑿 𝟏+𝑿 𝟐+𝑿 𝟑+⋯……+𝑿 𝒏 𝑵 = ∑ 𝑿 𝑵 Here 𝑋̅ = Arithmetic Means ∑ 𝑋 = Sum of all the values of the variable 𝑋 𝑁 = Number of the Observations Steps for finding Arithmetic mean : 1. Add together all the values of the variable 𝑋 and obtain the total i.e., ∑ 𝑋 2. Divide this total by the number of observations, i.e., 𝑁 Example-The following table data is the monthly income (in Rs.)of 10 employees in an office: 14780,15760,26690,27750,24840,24920,16100,17810,27050,26950 Calculate the arithmetic mean of incomes. Solution: Here 𝑁 = 10  ∴ 𝑋̅ = 𝑋1+𝑋2+𝑋3+⋯……+𝑋 𝑛 𝑁 = ∑ 𝑋 𝑁
  • 14. Unit-3 FREQUAENCY DISTRIBUTION RAI UNIVERSITY, AHMEDABAD 14  ∴ 𝑋̅ = 14780+15760+26690+27750+24840+24920+16100+17810+27050+26950 10  ∴ 𝑋̅ = 2,22,650 10  ∴ 𝑋̅ = 22,265  Hence the Average Income is Rs.22,265 4.2.2 Short-cut Method for finding Arithmetic Mean: The arithmetic mean can be calculated by using what is known as an arbitrary Origin. When deviations are taking from an arbitrary origin, the formula for calculating arithmetic mean is 𝑋̅ = 𝐴 + ∑ 𝑑 𝑁 Where A is the assumed mean and d is the deviation of items from assumed mean, i.e.,𝑑 = (𝑋 − 𝐴) Steps for finding Arithmetic mean by shortcut Method 1. Take an assumed mean. 2. Take the deviations of items from the assumed mean and denote these deviations by 𝑑. 3. Obtain the sum of these deviations, i.e., ∑ 𝑑 4. Apply the formula 𝑋̅ = 𝐴 + ∑ 𝑑 𝑁 Example-- The following table data is the monthly income (in Rs.)of 10 employees in an office: 14780,15760,26690,27750,24840,24920,16100,17810,27050,26950 Calculate the arithmetic mean of incomes by using short cut method. Solution: Supposeassumed mean 𝐴 = 22000
  • 15. Unit-3 FREQUAENCY DISTRIBUTION RAI UNIVERSITY, AHMEDABAD 15 Employees Income(Rs.) ( 𝑿 − 𝟐𝟐𝟎𝟎𝟎) = 𝒅 1 14,780 -7220 2 15,760 -6240 3 26,690 +4690 4 27,750 +5750 5 24,840 +2840 6 24,920 +2920 7 16,100 -5900 8 17,810 -4190 9 27,050 +5050 10 26,950 +4950 N=10 ∑ 𝑑 = 2650  𝑋̅ = 𝐴 + ∑ 𝑑 𝑁  𝐻𝑒𝑟𝑒 𝐴 = 22000,∑ 𝑑 = 2650, 𝑁 = 10  𝑋̅ = 22000 + 2650 10 = 22,265  Hence the average income is Rs. 22,265. 4.3 Calculationof Arithmetic Mean-DiscretefrequencyDistribution: In discrete series arithmetic mean may be computed by applying 1. Direct Method 2. Short-Cut Method 4.3.1DirectMethod: The formula for computing mean is
  • 16. Unit-3 FREQUAENCY DISTRIBUTION RAI UNIVERSITY, AHMEDABAD 16 𝑋̅ = ∑ 𝑓𝑥 𝑁 Where 𝑓 = frequency 𝑋 = The variable in Question 𝑁 = Total number of Observations i.e. ∑ 𝑓 Steps for finding Arithmetic Mean for Discrete frequency Distribution: 1. Multiply the frequency of each row with the variable and obtain the total ∑ 𝑓𝑋 2. Divide the total obtained by step(i) by the number of observations , i.e. Total frequency Example-- From the following data of the marks obtained by 60 students of a class calculatesthe arithmetic mean by Direct Method: Marks No. of Students 20 8 30 12 40 20 50 10 60 6 70 4 Solution: Let the marks be denoted by 𝑋 and the number of students by 𝑓. Calculation of Arithmetic Mean Marks(𝑿) No. of Students(𝒇) 𝒇𝑿 20 8 160
  • 17. Unit-3 FREQUAENCY DISTRIBUTION RAI UNIVERSITY, AHMEDABAD 17 30 12 360 40 20 800 50 10 500 60 6 360 70 4 280 𝑵 = 𝟔𝟎 ∑𝒇𝒙 = 𝟐𝟒𝟔𝟎  𝑋̅ = ∑ 𝑓𝑋 𝑁 = 2460 60 = 41 Hence, the average marks=41 4.3.2 Short-Cut Method: According to this method, 𝑋̅ = 𝐴 + ∑ 𝑓𝑑 𝑁 Where 𝐴 = Assumed mean ; 𝑑 = ( 𝑋 − 𝐴); 𝑁 = Total number of observations i.e., ∑ 𝑓. Steps for finding Arithmetic Mean for Discrete frequency Distribution: 1. Take an assumed mean. 2. Take the deviations of the variable X from the assumed mean and denote the deviations by 𝑑. 3. Multiply these deviations with the respective frequency and take the total ∑ 𝑓𝑑. 4. Divide the total obtained in third step by the total frequency. Example--From the following data of the marks obtained by 60 students of a class, calculatethe arithmetic mean by Shot-cut Method: Marks No. of Students 20 8
  • 18. Unit-3 FREQUAENCY DISTRIBUTION RAI UNIVERSITY, AHMEDABAD 18 30 12 40 20 50 10 60 6 70 4 Solution: Supposeassumed mean A= 40 Calculation of Arithmetic Mean by short-cut Method Marks(𝑿) No. of Students(𝒇) ( 𝑿 − 𝟒𝟎) = 𝒅 𝒇𝒅 20 8 -20 -160 30 12 -10 -120 40 20 0 0 50 10 +10 +100 60 6 +20 +120 70 4 +30 +120 𝑁 = 60 ∑ 𝑓𝑑 = 60  𝑋̅ = ∑ 𝑓𝑑 𝑁 = 40 + 60 60 = 40 + 1 = 41 Hence the Arithmetic mean by Shortcut method is =60  Note:We can Observe that value of Arithmetic mean does not change in both the method . so, we can use any one for finding arithmetic mean. 4.4 Calculationof Arithmetic Mean – Continuous Frequency Distribution
  • 19. Unit-3 FREQUAENCY DISTRIBUTION RAI UNIVERSITY, AHMEDABAD 19 In continuous frequency distribution arithmetic mean may be computed by applying any of the following methods: 1. Direct Method 2. Short-Cut Method 4.4.1 DirectMethod: (Meanof Continuous frequency distribution) When direct method is used 𝑿̅ = ∑ 𝒇𝒎 𝑵 Where, m = mid-point of various classes 𝑓 = Frequency of each class 𝑁 = The total frequency Steps for finding Arithmetic mean by DirectMethod (C.F.D): 1. Obtain the mid-point of each class and denote it by m. 2. Multiply these midpoints by the respective frequency of each class and obtain the total ∑ 𝑓𝑚 3. Divide the total obtained in step(i) by the sum of the frequency,i.e.,N. Example--.from the following data compute arithmetic mean by direct method: Marks 0-10 10-20 20-30 30-40 40-50 50-60 No. of Students 5 10 25 30 20 10 Solution: Calculation for Arithmetic Mean Marks(𝑿) Mid-points (𝒎) No. of Students (𝒇) 𝒇𝒎 0-10 5 5 25 10-20 15 10 150
  • 20. Unit-3 FREQUAENCY DISTRIBUTION RAI UNIVERSITY, AHMEDABAD 20 20-30 25 25 625 30-40 35 30 1050 40-50 45 20 900 50-60 55 10 550 𝑵 = 𝟏𝟎𝟎 ∑ 𝒇𝒎 = 𝟑, 𝟑𝟎𝟎  𝑋̅ = ∑ 𝑓𝑚 𝑁 = 3300 100 = 33 Hence The value of Arithmetic mean is 33. 4.4.2 Short-Cut Method: (Meanof Continuous frequency distribution) When short-cut method is used, arithmetic mean is computed by applying the following formula: 𝑋̅ = 𝐴 + ∑ 𝑓𝑑 𝑁 Where A= Assumed mean 𝑑 = Deviations of mid-points from assumed mean i.e.,(m-A) 𝑁 = Total number of observations Steps for finding Arithmetic Mean by Short-Cut Method: (C.F.D) 1. Take an assumed mean 2. From the mid-point of each class deductthe assumed mean. 3. Multiply the respective frequencies of each class by these deviations and obtain the total ∑ 𝑓𝑑. 4. Apply the formula 𝑋̅ = 𝐴 + ∑ 𝑓𝑑 𝑁 Example --from the following data compute arithmetic mean by Short-Cut method: Marks 0-10 10-20 20-30 30-40 40-50 50-60 No. of Students 5 10 25 30 20 10
  • 21. Unit-3 FREQUAENCY DISTRIBUTION RAI UNIVERSITY, AHMEDABAD 21 Solution: Assumed that Arithmetic Mean =35 N= 100 Calculation of Arithmetic mean by short-cut method Marks(𝑿) Mid-points (𝒎) No. of Students (𝒇) ( 𝒎 − 𝟑𝟓) = 𝒅 𝒇𝒅 0-10 5 5 -30 -150 10-20 15 10 -20 -200 20-30 25 25 -10 -250 30-40 35 30 0 0 40-50 45 20 +10 +200 50-60 55 10 +20 +200 𝑵 = 𝟏𝟎𝟎 ∑ 𝒇𝒅 = −𝟐𝟎𝟎  𝑋̅ = 𝐴 + ∑ 𝑓𝑑 𝑁 = 35 − 200 100 = 35 − 2 = 33 Hence the value of Arithmetic mean by Short-Cut method for continuous frequency distribution is 33. Example-- The mean marks of 100 students were found to be 40. Later on it was discoveredthat a score of 53 was misread as 83.Findthe correctmean corresponding to correctScore. Solution:  We are given 𝑁 = 100, 𝑋̅ = 40  Since 𝑋̅ = ∑ 𝑋 𝑁  ∑ 𝑋 = 𝑁𝑋̅ = 100× 40 = 4000  But this is not correct ∑ 𝑋
  • 22. Unit-3 FREQUAENCY DISTRIBUTION RAI UNIVERSITY, AHMEDABAD 22  Correct ∑ 𝑋 = Incorrect ∑ 𝑋 − Wrong item + Correct item  Correct ∑ 𝑋 = 4000− 83 + 53 = 3970  ∴ Correct 𝑋̅ = correct ∑ 𝑋 N = 3970 100 = 39.7 Hence the Correct mean=39.7 4.5 Weightedarithmetic mean One of the limitations of the arithmetic mean discussed above is that it gives equal importance to all the items. But there are cases where the relative importance of the different items.The formula for computing weighted arithmetic mean is: 𝑋 𝑤 ̅̅̅̅ = ∑ 𝑊𝑋 ∑ 𝑊  Where 𝑋 𝑤 ̅̅̅̅ represents the weighted arithmetic mean; X represents the variable values. W represents the weights attached to variable values. Steps for finding Weighted mean: 1. Multiply the weights by the variable X and obtain the total ∑ 𝑊𝑋 2. Divide this total by the sum of the weights, i.e.,∑ 𝑊. Example-- Calculate weightedaverage ofthe following data: Course BA BSc MA MCA MBA %of Pass 70 65 75 90 99 No of Students 20 30 30 50 40 Solution:  Weighted Average = ∑ 𝑊𝑋 ∑ 𝑊  Weighted Average =13300 / 170 = 76.47 Calculation for weighted Average (Mean)
  • 23. Unit-3 FREQUAENCY DISTRIBUTION RAI UNIVERSITY, AHMEDABAD 23 % of Pass X No of Student W XW 70 20 1400 65 30 1950 75 30 2250 90 50 4500 80 40 3200 X=380 W=170 13300 Example--A train runs 25 miles at a speedof 30 m.p.h., another50 miles at a speedof 40 m.p.h, then due to repairs a track travels for 6 minutes at a speed of 10 m.p.h. what is the average speedin miles per hour? Solution: Time taken in covering 25 miles at a speed of 30 m.p.h=50 minutes. Time taken in covering 50 miles at a speed of 40 m.p.h = 75 minutes. Distance covered in 6 minutes at a speed of 10 m.p.h = 1 mile. Time taken in covering 24 miles at a speed of 24 m.p.h = 60 minute Therefore, taking the time taken as weights we have the weighted mean as Speed (in m.p.h) (𝑿) Time taken (in min) W 𝑾𝑿 30 50 1500 40 75 3000 10 6 60 24 60 1440 ∑ 𝑊 = 191 ∑ 𝑊 𝑋 = 6000  ∴ Weighted arithmetic mean =𝑋 𝑤 ̅̅̅̅ = ∑ 𝑊𝑋 ∑ 𝑊 = 6000 191 = 31.41 𝑚. 𝑝. ℎ.
  • 24. Unit-3 FREQUAENCY DISTRIBUTION RAI UNIVERSITY, AHMEDABAD 24 Hence value of weighted arithmetic mean is 31.41 𝑚. 𝑝. ℎ. 5.1 MathematicalProperties Of Arithmetic Mean: The following are a few important mathematical properties of the arithmetic mean: 1. The sum of the deviations of the items from the arithmetic mean (taking signs into account) is always zero. i.e. ∑(𝑋 − 𝑋̅) = 0 2. The sum of the squared deviations of the items from arithmetic mean is minimum, that is, less than the sum of the squared deviations of the items from any other value. 3. It is clear that ∑(𝑋 − 𝑋̅)2 is greater. This Property that the sum of the squares of items is least from the mean is of immense use regression analysis which shall be discussed later. 5.2 Merits of arithmetic mean: Arithmetic mean is the simplest measurement of central tendency of a group. It is extensively used because:  It is easy to calculate and easy to understand.  It is based on all the observations.  It is rigidly defined.  It provides good basis of comparison.  It can be used for further analysis and algebraic treatment. 5.3 Demerits of the arithmetic mean:  It is affected by the extreme values.  It may lead to a wrong conclusion.  It is unrealistic.  Arithmetic mean cannot be obtained even if single observation is missing  It cannot be identified observation or graphic method 6.1Median:
  • 25. Unit-3 FREQUAENCY DISTRIBUTION RAI UNIVERSITY, AHMEDABAD 25 The median is the middle score for a set of data that has been arranged in order of magnitude. If the number of events are even then the average of two middle are taken. The median is better for describing the typical value. Example: In order to calculate the median, supposewe have the data below: 65 55 89 56 35 14 56 55 87 45 92 We first need to rearrange that data into order of magnitude (smallest first): 14 35 45 55 55 56 56 65 87 89 92 Our median mark is the middle mark - in this case, 56 (highlighted in bold). The median by definition refers to the middle value in a distribution. The median is that value of the series which divides the group into two equal parts, one part comprising all values greater than the median value and the other part comprising all the values smaller than the median value. 6.2 Steps for Calculationof Median – Individual Observations 1. Arrange the data in ascending or descending order of magnitude. (Both arrangements would give the same answer) 2. Apply the formula Median = 𝑀 = 𝑁+1 2 Example-- Find the median for the following data: 5, 15, 10, 15, 5, 10, 10, 20, 25 and 15. Solution: First of all we have to arrange all the Observations in ascending order 5, 5, 10, 10, 10, 15, 15, 15, 20, 25 Here by Observation we can say that 𝑁 = 10 𝐻𝑒𝑛𝑐𝑒, Median 𝑀 = 𝑁+1 2 = 10+1 2 = 5.5th item = 10+15 2 = 12.5 Example-- Find the median for the following data:
  • 26. Unit-3 FREQUAENCY DISTRIBUTION RAI UNIVERSITY, AHMEDABAD 26 25900,26950,27020,27200,28280 Solution: First of all we have to arrange all the Observations in ascending order 25900, 26950, 27020, 27200, 28280 Here by Observation we can say that 𝑁 = 5 𝐻𝑒𝑛𝑐𝑒, Median 𝑀 = 𝑁+1 2 = 5+1 2 = 6 2 = 3𝑟𝑑 item =27020 6.3 Steps for Calculationof median of continuous frequency distribution: 1. Determine the particular class in which the value of median lies. 2. Use 𝑁 2 as the rank of the median and not 𝑁+1 2 . 3. After ascertaining the class in which median lies, the following formula is used for determining the exact value of median 𝑀𝑒𝑑𝑖𝑎𝑛 = 𝐿 + 𝑁 2 − 𝑐. 𝑓. 𝑓 × 𝑖 Where, 𝐿 = Lower limit of the median class i.e., the class in which the middle item of the distribution lies. 𝑐. 𝑓. = Cumulative frequency of the class preceding the median class or sum of the Frequency of the frequencies of all classes lower than the median class. 𝑓 = Simple frequency of the Median class. 𝑖 = The class interval of the Median class. Example--: Find the median of the following data. Cost 10-20 20-30 30-40 40-50 50-60 Items in a group 4 5 3 6 3 Solution:
  • 27. Unit-3 FREQUAENCY DISTRIBUTION RAI UNIVERSITY, AHMEDABAD 27 Calculation for Median Cost Number of items in the group Cumulative frequency 10-20 4 4 20-30 5 9 30-40 3 12 40-50 6 18 50-60 3 21  Here N=21 ⇒ 𝑁 2 = 10.5  The median class is 30-40. From Formula, 𝑀𝑒𝑑𝑖𝑎𝑛 = 𝐿 + ( 𝑁 2 − 𝑐𝑓 𝑓 ) × 𝑖  Here, L=30, 𝑖 = 10, 𝑐𝑓 = 9  𝑀𝑒𝑑𝑖𝑎𝑛 = 30+ (10.5−9) 12 × 10 = 30 + 1.25 = 31.25 7.1 MathematicalProperty of Median: 1. The sum of the deviations of the items from median, ignoring signs, is the least. For example, the median of 4,6,8,10,12 is 8. The deviations from 8 ignoring signs are 4, 2, 0, 2, 4 and the total is 12.This total is smaller than the one obtained if deviations are taken from any other value. Thus if deviations are taken from 7, values ignoring signs would be 3, 1, 1, 3, 5 and the total 13. 7.2 Merits of Median:  It is easy to calculate and easy to understand.  It is based on all the observations.  It is rigidly defined.
  • 28. Unit-3 FREQUAENCY DISTRIBUTION RAI UNIVERSITY, AHMEDABAD 28  It eliminates the impact of extreme values.  It can be used for further analysis and algebraic treatment.  Median can be found out just by inspection in some cases. 7.3Demerits Of Median:  It simply ignores the extreme values.  It may lead to a wrong conclusion. When distribution of observations is Irregular.  The median is estimated in continuous case. 8.1 Mode: The value of the variable which occurs mostfrequently in a distribution is called the mode. Mode = 3 Median – 2 Mean 8.2 Calculationof Mode – Individual Operations. For determining mode count the number of times the various values repeat themselves and the value occurring maximum number of times is the mdal value .The more often the modal value appears relatively,the more valuable the measure is an average to represent data. Example--The following is the number of problems that Ms. Matty assigned for homework on 10 different days. What is the mode? 8, 11, 9, 14, 9, 15, 18, 6, 9, 10 Solution: Ordering the data from least to greatest, we get: 6, 8, 9, 9, 9, 10, 11, 14, 15, 18 The scorewhich occurs mostoften is 9. Therefore, the mode is 9.
  • 29. Unit-3 FREQUAENCY DISTRIBUTION RAI UNIVERSITY, AHMEDABAD 29 Example-2 In a crashtest, 11 cars were testedto determine what impact speed was required to obtain minimal bumper damage. Find the mode of the speeds given in miles per hour below. 24, 15, 18, 20, 18, 22, 24, 26, 18, 26, 24 Solution: Ordering the data from least to greatest, we get: 15, 18, 18, 18, 20, 22, 24, 24, 24, 26, 26 Since both 18 and 24 occurthree times, the modes are 18 and 24 miles per hour. This data set is bimodal. Example-3 A marathon race was completed by 5 participants. What is the mode of these times given in hours? 2.7 hr, 8.3 hr, 3.5 hr, 5.1 hr, 4.9 hr Solution: Ordering the data from least to greatest, we get: 2.7, 3.5, 4.9, 5.1, 8.3 Since each value occurs only once in the data set, there is no mode for this set of data. 8.2Steps for Calculationof Mode For Continuous frequency Distribution: 1. Constructthe table
  • 30. Unit-3 FREQUAENCY DISTRIBUTION RAI UNIVERSITY, AHMEDABAD 30 2. Find the Modal class 3. Find out Mode class by using N / 2 4. Apply the formula 𝑀𝑜𝑑𝑒 = 𝐿 + (𝑓1 − 𝑓0) 2𝑓1 − 𝑓0 − 𝑓2 × 𝑖 Where, L = Lower limit of the Modal class 𝑓1Frequency of the Modal class 𝑓0 = Frequency of the class preceding the Modal class 𝑓2 =Frequency of the class succeeding Modal class i = Class interval of modal class Example--: Calculate mode of the following data: Marks 10-20 20-30 30-40 40-50 50-60 F 5 20 25 15 5 Solution: Construction of the table to find Mode: Marks F 10-20 5 20-30 20 30-40 25 40-50 15 50-60 5 Modal class is 30-40 since highest frequency occurs here i.e. frequency of that class is =25 𝑀𝑜𝑑𝑒 = 𝐿 + (𝑓1 − 𝑓0) 2𝑓1 − 𝑓0 − 𝑓2 × 𝑖 Where, L = Lower limit of the Modal class = 30
  • 31. Unit-3 FREQUAENCY DISTRIBUTION RAI UNIVERSITY, AHMEDABAD 31 𝑓1 = Frequency of the Modal class = 25 𝑓0 = Frequency of the class preceding the Modal class = 20 𝑓2 =Frequency of the class succeeding Modal class =15 i = Class interval of modal class =10  𝑀𝑜𝑑𝑒 = 30+ (25−20) (2×25)−20−15 × 10  𝑀𝑜𝑑𝑒 = 30+ 50 15  𝑀𝑜𝑑𝑒 = 33.33 9.1Merits of Mode:  It is easy to calculate and easy to understand.  It eliminates the impact of extreme values  It can be identified by using graphical method 9.2Demerits of Mode  It is not suitable for further mathematical treatments.  It may lead to a wrong conclusion. When bimodal distribution.  It is difficult to compute in some cases.  Mode is influenced by length of the class interval. 10.1 Introduction Averages give us information of concentration of the observations about the central part of the distribution. But they fail to give anything further about the data. According to George Simpson and Fritz Kafka, “An average does not tell the full Story. It is hardly fully representative of a mass, unless we know the manner in which the individual items scatter around it. “Dispersion is the measure of the variations of the items.’’- A.L Bowley 10.2 RANGE The range is the difference between two extreme values of the given observations Range = Largest value – Smallest value
  • 32. Unit-3 FREQUAENCY DISTRIBUTION RAI UNIVERSITY, AHMEDABAD 32 10.3 Co-efficientof Range Co- efficient of Range = 𝐿𝑎𝑟𝑔𝑒𝑠𝑡 𝑣𝑎𝑙𝑢𝑒−𝑆𝑚𝑎𝑙𝑙𝑒𝑠𝑡 𝑣𝑎𝑙𝑢𝑒 𝐿𝑎𝑟𝑔𝑒𝑠𝑡 𝑣𝑎𝑙𝑢𝑒+𝑆𝑚𝑎𝑙𝑙𝑒𝑠𝑡 𝑣𝑎𝑙𝑢𝑒 = 𝐿−𝑆 𝐿+𝑆 Example-- Find the Co-efficient of range of Marks of 10 students from the following 65,35,48,99,56,88,78,20,66,53 Solution:  Range = L – S  Range = 99 -20  Range = 79  Co- efficient of Range = 99−20 99+20 = 79 109 = 0.72 10.4 Merits: 1. It is easy to compute and understand. 2. It gives an idea about the distribution immediately. 10.5 Demerits: 1. Calculation range depends only on the basis of extreme items, hence it is not reliable. 2. It is not applied to open end cases 3. Not suitable for mathematical treatments. 11.1Quartile Deviation: The range which includes the middle 50 per cent of the distribution.That is one quarter of the observations at the lower end, another quarter of the observations at the upper end of the distribution are excluded in computing the interquartile range.In the other words, interquartile range represents the difference between the third quartile and the first quartile. Symbolically,
  • 33. Unit-3 FREQUAENCY DISTRIBUTION RAI UNIVERSITY, AHMEDABAD 33 𝐼𝑛𝑡𝑒𝑟𝑞𝑢𝑎𝑟𝑡𝑖𝑙𝑒 𝑟𝑎𝑛𝑔𝑒 = 𝑄3 − 𝑄1 Very often the interquartile range is reduced to the form of the semi-interquartile range or quartile deviation by dividing it by 2. 𝑄𝑢𝑎𝑟𝑡𝑖𝑙𝑒 𝐷𝑒𝑣𝑖𝑎𝑡𝑖𝑜𝑛 𝑜𝑟 𝑄. 𝐷. = 𝑄3 − 𝑄1 2 11.2 Coefficientof Quartile Deviation: Quartile deviation is an absolute measure of dispersion .The relative measure corresponding to this measure, called the coefficient of quartile deviation, is calculated as follows. 𝐶𝑜𝑒𝑓𝑓𝑖𝑐𝑖𝑒𝑛𝑡 𝑜𝑓 𝑄. 𝐷. = 𝑄3 − 𝑄1 2 𝑄3 + 𝑄1 2 = 𝑄3 − 𝑄1 𝑄3 + 𝑄1 Example--find out the value of quartile deviation and its coefficientfrom the following data: Roll No. 1 2 3 4 5 6 7 Marks 20 28 40 12 30 15 50 Solution:  Marks arranged in ascending order: 12 15 20 28 30 40 50  𝑄1 = Size of 𝑁+1 4 th item = 7+1 4 = 2𝑛𝑑 item  Size of 2nd item is 15. Thus 𝑄1 = 15  𝑄3 = size of 3 ( 𝑁+1 4 ) 𝑡ℎ item = Size of ( 3×8 4 ) 𝑡ℎ item =6th item  Size of 6th item is 40.Thus 𝑄3 = 40 𝑄. 𝐷. = 𝑄3 − 𝑄1 2 = 40 − 15 2 = 12.  Now we have to find coefficient of Q.D. 𝐶𝑜𝑒𝑓𝑓𝑖𝑐𝑖𝑒𝑛𝑡 𝑜𝑓 𝑄. 𝐷. = 𝑄3−𝑄1 𝑄3+𝑄1 = 40−15 40+15 = 25 55 = 0.455
  • 34. Unit-3 FREQUAENCY DISTRIBUTION RAI UNIVERSITY, AHMEDABAD 34 Example--Compute coefficientnofnquartile deviation from the following data: Marks 10 20 30 40 50 60 No. of Students 4 7 15 8 7 2 Solution: Calculation of coefficient of quartile deviation Marks Frequency c.f. 10 4 4 20 7 11 30 15 26 40 8 34 50 7 41 60 2 43  𝑄1 = size of 𝑁+1 4 𝑡ℎ item = 43+1 4 = 11th item  Size of 11th item is 20. Thus 𝑄1 = 20  𝑄3 = Size of 3 ( 𝑁+1 4 ) 𝑡ℎ item = 3×44 4 = 33rd item  Size of 33rd item is 40. Thus, 𝑄3 = 40 𝑄. 𝐷. = 𝑄3−𝑄1 2 = 40−20 2 = 10  𝐶𝑜𝑒𝑓𝑓𝑖𝑐𝑖𝑒𝑛𝑡 𝑜𝑓 𝑄. 𝐷. = 𝑄3−𝑄1 𝑄3+𝑄1 = 40−20 40+20 = 0.333 12.1MeanDeviation: Mean deviation is the arithmetic mean of the difference of a series computed from any measure of central tendency i.e., Deviations from Mean or Mode or Median. All the deviation’s absolute values are considered. The mean deviation is also known as the average deviation. It is the average difference between the items in a distribution and the median or mean of that
  • 35. Unit-3 FREQUAENCY DISTRIBUTION RAI UNIVERSITY, AHMEDABAD 35 series. Theoritically there is an advantage in taking the deviations from median because the sum of deviations of items from median is minimum when signs are ignored. 12.2Computation of MeanDeviation-Individual Observations: If 𝑋1, 𝑋2, 𝑋3, …… 𝑋 𝑁 are 𝑁 given observations then the deviation about an average 𝐴 is given by 𝑀. 𝐷. = 1 𝑁 ∑| 𝑋 − 𝐴| = 1 𝑁 ∑| 𝐷| 𝑜𝑟 ∑| 𝐷| 𝑁 Where | 𝐷| = | 𝑋 − 𝐴|. Read as mod (X-A) is the modulus value or absolute value of the deviation ignoring plus and minus signs. 12.2 Steps for Computation of mean deviation: (Indiviadual Observations) 1. Compute the median of the series. 2. The deviations of items from median ignoring ± signs and denote these deviations by | 𝐷|. 3. Obtain the total of these deviations,ie. ∑| 𝐷|. 4. Divide the total obntained in step (3) by the total number of observations. 12.3Coefficientof MeanDeviation: The relative measure corresponding to the mean deviation called the coefficient of mean deviation is obtained by dividing mean deviation by the particular average used in computing mean deviation. Thus if mean deviation has been computed from median, the coefficient of mean deviation shall be obtained by dividing mean deviation by mean, median or mode. 𝐶𝑜𝑒𝑓𝑓𝑖𝑐𝑖𝑒𝑛𝑡 𝑜𝑓 𝑀. 𝐷. = 𝑀.𝐷. 𝑀𝑒𝑎𝑛 ,𝑀𝑒𝑑𝑖𝑎𝑛 𝑜𝑟 𝑀𝑜𝑑𝑒 Example--Calculate mean deviation and coefficient of mean deviation from the following data:
  • 36. Unit-3 FREQUAENCY DISTRIBUTION RAI UNIVERSITY, AHMEDABAD 36 100 200 300 400 500 600 700 Solution: Calculation for Mean Deviation 𝑿 | 𝑫| = | 𝑿 − 𝑨|=| 𝑿 − 𝟒𝟎𝟎| 100 300 200 200 300 100 400 0 500 100 600 200 700 300 ∑| 𝑫| = 𝟏𝟐𝟎𝟎  Arithmetic Mean=A = ∑ 𝑋𝑖 𝑁 = 2800 7 = 400  Mean Deviation= 𝑀. 𝐷. = 1 𝑁 ∑| 𝑋 − 𝐴| = 1 𝑁 ∑| 𝐷| 𝑀. 𝐷. = 1200 7 = 171.42  𝐶𝑜𝑒𝑓𝑓𝑖𝑐𝑖𝑒𝑛𝑡 𝑜𝑓 𝑀. 𝐷. = 𝑀.𝐷. 𝑀𝑒𝑎𝑛 = 171.42 400 = 0.4285  𝐶𝑜𝑒𝑓𝑓𝑖𝑐𝑖𝑒𝑛𝑡 𝑜𝑓 𝑀. 𝐷 = 0.43 12.2.2 Calculationof MeanDeviation-(Discrete frequency distribution): In discrete series the formula for calculating mean deviation is 𝑀. 𝐷. = ∑ 𝑓| 𝐷| 𝑁 Where | 𝐷| denote deviation from median ignoring signs.
  • 37. Unit-3 FREQUAENCY DISTRIBUTION RAI UNIVERSITY, AHMEDABAD 37 Steps for calculation: 1. Calculate the median of the series. 2. Take the deviations of the items from median ignoring signs and denote them by | 𝐷|. 3. Divide the total obtained in step (ii) by the number of observations.This gives us the value of mean deviation. Example--Claculate Meandeviation from the following series X 10 11 12 13 14 f 3 12 18 12 3 Solution: Calculation of Mean Deviation X 𝒇 | 𝑫| 𝒇| 𝑫| 𝒄. 𝒇. 10 3 2 6 3 11 12 1 12 15 12 18 0 0 33 13 12 1 12 45 14 3 2 6 48 𝑁 = 48 ∑ 𝑓| 𝐷|=36  𝑀. 𝐷.= ∑ 𝑓| 𝐷| 𝑁  Median = Size of 𝑁+1 2 𝑡ℎ item = 48+1 2 = 24.5𝑡ℎ item  Size of 24.5th item is 12, hence Median = 12  𝑀. 𝐷. = ∑ 𝑓| 𝐷| 𝑁 = 36 48 = 0.75 12.2.3 Calculationof MeanDeviation– Continuous Frequency distribution
  • 38. Unit-3 FREQUAENCY DISTRIBUTION RAI UNIVERSITY, AHMEDABAD 38 For calculating mean deviation in continuous series the procedureremains the same as discussed above.Theonly difference is that here we have to obtain the mid point of the various classes and take deviations of these points from median. The formula is same, i.e., 𝑀. 𝐷. = ∑ 𝑓| 𝐷| 𝑁 Example-- Calculate the mean deviation from mean for the following data: Class Interval 2-4 4-6 6-8 8-10 frequency 6 8 4 2 Solution: Calculation for Mean Deviation Class Mid value(m) Frequency 𝒇𝒎 | 𝑫| = | 𝑿 − 𝑨| 𝒇| 𝑫| 2-4 3 6 18 2.2 13.2 4-6 5 8 40 0.2 1.6 6-8 7 4 28 1.8 7.2 8-10 9 2 18 3.8 7.6 Total 20 104 27.6  𝐴 = ∑ 𝑓𝑚 ∑ 𝑓 = 104 20 = 5.2  𝑀. 𝐷. = ∑ 𝑓| 𝐷| 𝑁 = 27.6 20 = 1.48 (ℎ𝑒𝑟𝑒 𝑁 = ∑ 𝑓 = 20)  𝑀. 𝐷. = 1.48 12.4Merits of MeanDeviation: 1. It is simple to understand and easy to calculate 2. The computation process is based on all items of the series
  • 39. Unit-3 FREQUAENCY DISTRIBUTION RAI UNIVERSITY, AHMEDABAD 39 3. It is less affected by the extreme items. 4. This measure is flexible, Since it can be calculated from mean, meadian, or mode. 5. This measure is rigidly defined. 12.5Demerits of Mean Deviation: 1. This measure is not a very accurate measure of dispersion. 2. Not suitable for further mathematical calculation. 3. It is rarely used. 4. Absolute values are considered, mathematically unsound and illogical. 13.1 Standard Deviation: The famous statistician karl pearsonintroduced the conceptof standard deviation in 18 This is the most accepted measure of dispersion and also widely used in many statistical applications. Standard deviation is also referred as root-mean square deviation or Mean square error. It gives accurate results. The standard deviation is also denoted by the greek letter ( 𝜎). 13.2Variance: The term variance was used to describe the square of standard deviation by R.A.Fisher in 1913. The conceptof variance is highly important in advanced work where it is possible to split the total into several parts,each attributable to one of the factors causing variation in their original series. Variance is defined as follows 𝑉𝑎𝑟𝑖𝑎𝑛𝑐𝑒 = ∑( 𝑋−𝐴)2 𝑁 Thus variance is nothing but the square of the standard deviation 𝑖. 𝑒., 𝑉𝑎𝑟𝑖𝑎𝑛𝑐𝑒 = 𝜎2 𝑜𝑟 𝜎 = √ 𝑉𝑎𝑟𝑖𝑎𝑛𝑐𝑒
  • 40. Unit-3 FREQUAENCY DISTRIBUTION RAI UNIVERSITY, AHMEDABAD 40 In a frequency distribution where deviations are taken from assumed mean variance may directly be computed as follows 𝑉𝑎𝑟𝑖𝑎𝑛𝑐𝑒 = { ∑ 𝑓𝑑2 𝑁 − ( ∑ 𝑓𝑑 𝑁 ) 2 } × 𝑖2 𝑤ℎ𝑒𝑟𝑒, 𝑑 = 𝑋−𝐴 𝑖 and 𝑖=class interval 13.2. CalculationOfStandard Deviation- Individual Observation There are two method of calculating standard deviation in an individual observation: (i) Direct Method – Deviation taken from actual mean (ii) Short- cut Method – Deviation taken from assumed mean 13.2.1 (i) DirectMethod: The following are the steps: 1. Find out actual mean of the given observations. 2. Compute deviation of each observation from the mean (𝑋 − 𝑀𝑒𝑎𝑛). 3. Square the deviations and find out the sum i.e. ( 𝑋 − 𝐴)2 4. Divide the total by the number of observations and take square rootof the quotient,the value is standard deviation. 𝜎 = √ ∑( 𝑋 − 𝐴)2 𝑁 Example-- Calculate the standard deviation from the following data: 𝟏𝟓, 𝟏𝟐, 𝟏𝟕, 𝟏𝟎, 𝟐𝟏, 𝟏𝟖, 𝟏𝟏, 𝟏𝟔 Solution: Calculation of S.D. from Mean Values (𝑿) (𝑿 − 𝑨) ( 𝑿 − 𝑨) 𝟐
  • 41. Unit-3 FREQUAENCY DISTRIBUTION RAI UNIVERSITY, AHMEDABAD 41 15 0 0 12 -3 9 17 2 4 10 -5 25 21 6 36 18 3 9 11 -4 16 16 1 1 𝑿 = 𝟏𝟐𝟎 ( 𝑿 − 𝑨) 𝟐 = 𝟏𝟎𝟎  𝐴 = ∑ 𝑋𝑖 𝑁 = 120 8 = 15  S.D.=𝜎 = √ ∑( 𝑋−𝐴)2 𝑁 = √ 100 8  𝜎 = 3.53  𝑣𝑎𝑟𝑖𝑎𝑛𝑐𝑒 = 𝜎2 = 12.46 13.2.2(ii) Short-Cut Method - Deviation takenfrom assumedmean This method is used when arithmetic mean is fractional value. A deviation from fractional value leads to tedious task. To save calculation time,we apply this method the formula is 𝜎 = √ ∑ 𝑑2 𝑁 − ( ∑ 𝑑 𝑁 ) 2 Where, d = deviations from assumed mean =(𝑋 − 𝐴) 𝑁 = Number of Observations Steps for calculations: 1. Take the deviations of the items from an assumed mean, i.e. obtain ( 𝑋 − 𝐴). Denote these deviations by d. 2. Take the total of these deviations. i.e., obtain ∑ 𝑑.
  • 42. Unit-3 FREQUAENCY DISTRIBUTION RAI UNIVERSITY, AHMEDABAD 42 3. Square these deviations and obtain the total ∑ 𝑑2 . 4. Substitute the values of ∑ 𝑑2 ,∑ 𝑑 𝑎𝑛𝑑 𝑁 in the above formula. Example--Bloodserum cholesterollevels of10 persons are as under: 240, 260, 290,245, 255, 288,272, 263, 277,251 Calculate standard deviation and variance with the help of assumed mean. Solution: Calculation of Standard Deviation 𝑿 𝒅 = (𝑿 − 𝟐𝟔𝟒) 𝒅 𝟐 240 -24 576 260 -4 16 290 +26 676 245 -19 361 255 -9 81 288 +24 576 272 +8 64 263 -1 1 277 +13 169 251 -13 169 ∑ 𝑿 =2641 ∑ 𝑑 = +1 ∑ 𝑑2 = 2689  𝜎 = √ ∑ 𝑑2 𝑁 − ( ∑ 𝑑 𝑁 ) 2  Now ∑ 𝑑2 = 2689,∑ 𝑑 = +1, 𝑁 = 10
  • 43. Unit-3 FREQUAENCY DISTRIBUTION RAI UNIVERSITY, AHMEDABAD 43  ∴ 𝜎 = √2689 10 − ( 1 10 ) 2  ∴ 𝜎 = √268.9− 0.01  ∴ 𝜎 = 16.398  𝑉𝑎𝑟𝑖𝑎𝑛𝑐𝑒 = 𝜎2 = (16.398)2 = 268.894 13.3 Calculationof Standard Deviation– Discrete frequency distribution For calculating standard deviation in discrete series, any of the following methods may be applied: 1. Actual Mean Method 2. Assumed Mean Method 3. Step deviation method 13.3.1 ActualMeanMethod: When this method is applied, deviations are taken from the actual mean,The formula is applied 𝜎 = √ ∑ 𝑓( 𝑋 − 𝐴)2 𝑁 However in practice this method is rarely used becauseif the actual mean is in fraction the calculations take a lot of time. Steps for calculation: (1) Compute mean of the observations (2) Compute deviation from the mean 𝑑 = ( 𝑋 − 𝐴) (3) Square the deviations d2 and multiply these values with respective frequencies f i.e., fd2 (4) Sum the products 𝑓𝑑2 and apply the formula 𝜎 = √ ∑ 𝑓𝑑2 𝑁 = √ ∑ 𝑓( 𝑋 − 𝐴)2 𝑁
  • 44. Unit-3 FREQUAENCY DISTRIBUTION RAI UNIVERSITY, AHMEDABAD 44 Example-- Compute standard deviation and variance from the following data Marks 10 20 30 40 50 Frequency 2 8 10 8 2 Solution: Construct the table to compute the standard deviation Marks (X) 𝒇 𝒇𝑿 𝒅 = 𝑿 − 𝟑𝟎 𝒅 𝟐 𝒇𝒅 𝟐 10 2 20 -20 400 800 20 8 160 -10 100 800 30 10 300 0 0 0 40 8 320 10 100 800 50 2 100 20 400 800 𝑓 = 30 𝑓𝑋 = 900 𝑓𝑑2 = 3200  Mean = 𝐴 = ∑ 𝑋𝑖 𝑁 = 900 30 = 30  𝜎 = √ ∑ 𝑓𝑑2 𝑁 = √ ∑ 𝑓( 𝑋−𝐴)2 𝑁  𝜎 = √ 3200 30  𝜎 = √106.66 = 10.325  𝑉𝑎𝑟𝑖𝑎𝑛𝑐𝑒 = 𝜎2 = 106.66 13.3.2 AssumedMeanMethod: When this method is used, the following formula is applied: 𝜎 = √ ∑ 𝑓𝑑2 𝑁 − ( ∑ 𝑓𝑑 𝑁 ) 2 Where 𝑑 = (𝑋 − 𝐴) Steps for calculation: 1. Assume any one of the given value as assumed mean A
  • 45. Unit-3 FREQUAENCY DISTRIBUTION RAI UNIVERSITY, AHMEDABAD 45 2. Compute deviation from the assumed mean (𝑑 = 𝑋 − 𝐴). 3. Multiply these deviations by its frequencies 𝑓𝑑. 4. Square the deviations (𝑑2 ) and multiply these values with respective Frequencies ( 𝑓) i.e., 𝑓𝑑2 5. Sum the products 𝑓𝑑2 and apply the formula 𝜎 = √ ∑ 𝑓𝑑2 𝑁 − ( ∑ 𝑓𝑑 𝑁 ) 2 Example-- Compute standard deviation and variance from the following data Marks 10 20 30 40 50 Frequency 2 8 10 8 2 Solution: Constructthe table to compute the standard deviation Marks (X) 𝒇 𝒅 = 𝑿 − 𝟐𝟎 𝒅 𝟐 𝒇𝒅 𝒇𝒅 𝟐 10 2 -10 100 -20 200 20 8 0 0 0 0 30 10 10 100 100 1000 40 8 20 400 160 3200 50 2 30 900 60 1800 𝑓 = 30 𝑓𝑑 = 300 𝑓𝑑2 = 6200  𝜎 = √ ∑ 𝑓𝑑2 𝑁 − ( ∑ 𝑓𝑑 𝑁 ) 2  𝑁𝑜𝑤, here ∑ 𝑓𝑑2 = 6200,∑ 𝑓𝑑 = 300, 𝑁 = ∑ 𝑓 = 30  𝜎 = √6200 30 − ( 300 30 ) 2  𝜎 = √206.66 − 100 = √106.66  𝜎 = 10.325
  • 46. Unit-3 FREQUAENCY DISTRIBUTION RAI UNIVERSITY, AHMEDABAD 46  𝑉𝑎𝑟𝑖𝑎𝑛𝑐𝑒 = 𝜎2 = 106.66 13.3.3StepdeviationMethod: When this method is used we take deviations of midpoints from an assumed mean and divide these deviations by the width of class interval,i.e. ′𝑖′ . In case class intervals are unequal. We divide the deviations of midpoints by the lowest common factor and use ‘c’ instead of ′𝑖′ in the formula for calculating standard deviation. The formula for calculating standard deviation is: 𝜎 = √ ∑ 𝑓𝑑2 𝑁 − ( ∑ 𝑓𝑑 𝑁 ) 2 × 𝑖 Where 𝑑= 𝑋−𝐴 𝑖 and 𝑖 = class interval Steps for calculation: Take a common factor and divide that item by all deviations 1. Assume any one of the given value as assumed mean A 2. Compute deviation from the assumed mean (𝑑 = 𝑋−𝐴 𝑖 ). 3. Multiply these deviations by its frequencies 𝑓𝑑. 4. Square the deviations (𝑑2 ) and multiply these values with respective Frequencies ( 𝑓) i.e., 𝑓𝑑2 5. Sum the products 𝑓𝑑2 and apply the formula. Example-- The annual salaries ofa group of employees are given in the following table: Salaries 45000 50000 55000 60000 65000 70000 75000 80000 Number of persons 3 5 8 7 9 7 4 7 Calculate the standard deviation of the salaries.
  • 47. Unit-3 FREQUAENCY DISTRIBUTION RAI UNIVERSITY, AHMEDABAD 47 Solution: Calculation of standard deviation Salaries (X) No. of persons(𝒇) 𝒅 = 𝑿 − 𝟔𝟎𝟎𝟎𝟎 𝟓𝟎𝟎𝟎 𝒇𝒅 𝒇𝒅 𝟐 45000 3 -3 -9 27 50000 5 -2 -10 20 55000 8 -1 -8 8 60000 7 0 0 0 65000 9 +1 +9 9 70000 7 +2 +14 28 75000 4 +3 +12 36 80000 7 +4 +28 112 𝑵 = 𝟓𝟎 ∑ 𝒇𝒅 =36 ∑ 𝒇𝒅 𝟐 = 𝟐𝟒𝟎  𝜎 = √ ∑ 𝑓𝑑2 𝑁 − ( ∑ 𝑓𝑑 𝑁 ) 2 × 𝑖  Now here ∑ 𝑓𝑑 = 36, ∑ 𝑓𝑑2 = 240 ,𝑁 = 50, 𝑖 = 5000  ∴ 𝜎 = √240 50 − ( 36 50 ) 2 × 5000  ∴ 𝜎 = √4.8− 0.5184× 5000  ∴ 𝜎 = 10346.01 13.4 Calculationof Standard Deviation– Continuous Series. In continuous series any of the methods discussed above for discrete frequency distribution can be used. However, in practice it is the step deviation method that is most used. The formula is,
  • 48. Unit-3 FREQUAENCY DISTRIBUTION RAI UNIVERSITY, AHMEDABAD 48 𝜎 = √ ∑ 𝑓𝑑2 𝑁 − ( ∑ 𝑓𝑑 𝑁 ) 2 × 𝑖 Where 𝑑 = 𝑚−𝐴 𝑖 , 𝑖 = 𝑐𝑙𝑎𝑠𝑠 𝑖𝑛𝑡𝑒𝑟𝑣𝑎𝑙 Steps for calculation: 1. Find the mid point of various classes. 2. Take the deviations of these mid points from an assumed mean and denote thrse deviations by 𝑑. 3. Wherever possible take a common factor and denote this column by 𝑑. 4. Multiply the frequencies of each class with these deviations and obtain ∑ 𝑓𝑑. 5. Square the deviations and multiply them with the respective frequencies of each class and obtain ∑ 𝑓𝑑2 . Thus the only difference in procedurein case of continuous series is to find mid- points of the various classes. Example--Calculate Standard deviation from the following data: Age 20-25 25-30 30-35 35-40 40-45 45-50 No. of Persons 170 110 80 45 40 35 Solution: Take assumed average = 32.5 Calculation of Standard deviation Age Mid point(m) No. of Persons (f) 𝒅 = 𝒎 − 𝟑𝟐. 𝟓 𝟓 𝒇𝒅 𝒇𝒅 𝟐 20-25 22.5 170 -2 -340 680 25-30 27.5 110 -1 -110 110 30-35 32.5 80 0 0 0 35-40 37.5 45 +1 +45 45
  • 49. Unit-3 FREQUAENCY DISTRIBUTION RAI UNIVERSITY, AHMEDABAD 49 40-45 42.5 40 +2 +80 160 45-50 47.5 35 +3 +105 315 𝑵 = 𝟒𝟖𝟎 ∑ 𝒇𝒅 =-220 ∑ 𝒇𝒅 𝟐 =1310  𝜎 = √ ∑ 𝑓𝑑2 𝑁 − ( ∑ 𝑓𝑑 𝑁 ) 2 × 𝑖  𝜎 = √1310 480 − ( −220 480 ) 2 × 5  𝜎 = √2.279− 0.21 × 5  𝜎 = √2.519× 5  𝜎 = 1.587× 5 = 7.936 14.1Coefficientof Variation: The standard deviation discussed aboveis an absolute measure of dispersion. The corresponding relative measure is known as the Coefficient of variation. This measure developed by karl pearson is the most commonly used measure of relative variation. It is used in such problems where we cant to compare the variability of two or more than two series. That series for which the coefficient of variation is greater is said to be more variable or conversely less consistent,less uniform,less srable . On the other hand , the series for which coefficient of variation is less is said to be less variable or more consistent,more uniform,more stable . Coefficient of variation is denoted by C.V. and is obtained as follows: 𝐶𝑜𝑒𝑓𝑓𝑖𝑐𝑖𝑒𝑛𝑡 𝑜𝑓 𝑣𝑎𝑟𝑖𝑎𝑡𝑖𝑜𝑛 𝑜𝑟 𝐶. 𝑉. = 𝜎 𝐴 × 100 Where, 𝜎 = standard deviation, 𝐴 = 𝑎𝑟𝑖𝑡ℎ𝑚𝑒𝑡𝑖𝑐 𝑀𝑒𝑎𝑛
  • 50. Unit-3 FREQUAENCY DISTRIBUTION RAI UNIVERSITY, AHMEDABAD 50 Example--From the prices of shares of 𝑿 and 𝒀 below find out which is more stable in value: 𝑿 35 54 52 53 56 58 52 50 51 49 𝒀 108 107 105 105 106 107 104 103 104 101 Solution: In order to find out which shares are more stable, we have to compare coefficient of variations. Calculation of Coefficient of Variation 𝑿 𝒙 = (𝑿 − 𝑨) 𝒙 𝟐 𝒀 𝒚 = (𝒀 − 𝑨) 𝒚 𝟐 35 -16 256 108 +3 9 54 +3 9 107 +2 4 52 +1 1 105 0 0 53 +2 4 105 0 0 56 +5 25 106 +1 1 58 +7 49 107 +2 4 52 +1 1 104 -1 1 50 -1 1 103 -2 4 51 0 0 104 -1 1 49 -2 4 101 -4 16 ∑ 𝑿 =510 ∑ 𝒙 =0 ∑ 𝒙 𝟐 =350 ∑ 𝒀 =1050 ∑ 𝒚=0 ∑ 𝒚 𝟐 =40 Coefficientof variation 𝑿:  𝐶. 𝑉. = 𝜎 𝐴 × 100  Here 𝐴 = ∑ 𝑋 𝑁 = 510 10 = 51
  • 51. Unit-3 FREQUAENCY DISTRIBUTION RAI UNIVERSITY, AHMEDABAD 51  𝜎 = √ ∑ 𝑥2 𝑁 = √ 350 10 = 5.916  ∴ 𝐶. 𝑉. = 𝜎 𝐴 × 100 = 5.196 51 × 100  ∴ 𝐶. 𝑉. = 11.6 Coefficientof variation 𝒀:  𝐶. 𝑉. = 𝜎 𝐴 × 100  Here 𝐴 = ∑ 𝑦 𝑁 = 1050 10 = 105  𝜎 = √ ∑ 𝑦2 𝑁 = √ 40 10 = 2  ∴ 𝐶. 𝑉. = 𝜎 𝐴 × 100 = 2 105 × 100  ∴ 𝐶. 𝑉. = 1.905 Since Coefficient of variation is much less in case of shares 𝑌, Hence they are more stable in value. EXERCISE Q-1 Evaluate the following Questions: (1). Find the Mean, Median and Mode of the following data. Cost 10-20 20-30 30-40 40-50 50-60 Items in a group 4 5 3 6 3 (2).Find the Mean, Median and Mode of the following distribution: Class Interval 0-10 10-20 20-30 30-40 40-50 50-60 60-70 70-80 Frequency 5 9 8 12 28 20 12 11 (3).On his first 5 biology tests, Bob received the following scores: 72, 86, 92, 63, and 77. What test scoremust Bob earn on his sixth test so that his average (mean score)for all six tests will be 80? Show how you arrived at your answer. Q-2 Evaluate the following Questions:
  • 52. Unit-3 FREQUAENCY DISTRIBUTION RAI UNIVERSITY, AHMEDABAD 52 (1). Calculate quartile deviation and it coefficient from the following data: Wages in Rupees per Day Less then 35 35- 37 38-40 41-43 Over 43 Number of Wage earners 14 62 99 18 7 (2). Calculate Mean Deviation for the following data Size 0-10 10-20 20-30 30-40 40-50 50-60 60-70 Frequency 7 12 18 25 16 14 8 (3). The owner of a restaurant is interested in how much people spend at the restaurant. He examines 10 randomly selected receipts for parties of four and write down the following data: 44, 50, 38, 96, 42, 47, 40,39, 46, 50. Find mean, standard deviation and variance. 15. Referenc Book andWebsite Name: 1. http://www.emathzone.com/tutorials/basic-statistics/collection-of-statistical- data.html 2. http://wizznotes.com/mathematics/statistics/class-limits-boundaries-and- intervals 3. http://tistats.com/definitions/class-width/ 4. http://mathespk.blogspot.in/2011/10/frequency-density.html 5. http://www.mathsisfun.com/definitions/relative-frequency.html 6. http://www.mathsisfun.com/definitions/cumulative-frequency.html 7. http://www.mathgoodies.com/lessons/vol8/mode.html 8. Statistical Methods by S.P.Gupta.
  • 53. Unit-3 FREQUAENCY DISTRIBUTION RAI UNIVERSITY, AHMEDABAD 53